1 Introduction

Let \(\displaystyle \Delta = \sum _{j=1}^n\frac{\partial ^2}{\partial x^2_j}\) be the Laplace operator in \(\mathbb {R}^n\). Consider its heat semigroup

$$\begin{aligned} e^{t\Delta }\varphi (x)=\int _{{\mathbb R}^n} W_t(x-y)\varphi (y){\textrm{d}}y,~x\in {\mathbb R}^{n},\ \ t>0, \end{aligned}$$

where W is the Gauss–Weierstrass kernel

$$\begin{aligned}W_t(x)=\frac{1}{(4\pi t)^{n/2}} e^{-\frac{|x|^2}{4t}}.\end{aligned}$$

For more information related with this semigroup, see [17].

For \(0<\alpha <1\), the generalized Poisson formula of f is given by

$$\begin{aligned} {\mathcal P}_t^\alpha f(x)&=\frac{t^{2\alpha }}{4^\alpha \Gamma (\alpha )} \int _0^{+\infty } e^{-t^2/(4s)}e^{s\Delta } f(x)\,\frac{{\textrm{d}}s}{s^{1+\alpha }}\nonumber \\&=\frac{t^{2\alpha }}{4^\alpha \Gamma (\alpha )} \int _0^{+\infty } \int _{{\mathbb R}^n} \frac{e^{-(t^2+|y|^2)/(4 s)}}{(4\pi s)^{n/2}} f (x-y){\textrm{d}}y \frac{{\textrm{d}}s}{s^{1+\alpha }}, \quad x\in {\mathbb R}^n, \ t>0. \end{aligned}$$
(1.1)

It means that the generalized Poisson formula can be obtained via the heat semigroup \(\displaystyle \{e^{t\Delta }\}_{t>0}\). In [6], Carffarelli and Silvestre studied the generalized Poisson formula to solve an extension problem. Stinga and Torrea defined this kind of Poisson formula for Hermite operator \(L=-\Delta +|x|^2\) in [19]. In the case \(\alpha =1/2\), \({\mathcal P}_t^{1/2}\) is the Bochner subordinated Poisson semigroup of \(e^{t\Delta }\); see [17].

Let \(\{a_j\}_{j\in {\mathbb Z}}\) be an increasing sequence of positive real numbers, and \(\{v_j\}_{j\in {\mathbb Z}}\) be a bounded sequence of real or complex numbers. Let \(\{T_t\}_{t>0}\) be an operator sequence. We consider the differential transform series

$$\begin{aligned} \sum _{j\in {\mathbb Z}} v_j(T_{a_{j+1}} f(x)-T_{a_j} f(x)). \end{aligned}$$
(1.2)

In [12], Jones and Rosenblatt studied the behavior of the series of the differences of ergodic averages and the differences of differentiation operators along lacunary sequences in the context of the \(L^p\) spaces. In [2], the authors solved these problems with a different approach, which relied heavily on the method of Calderón–Zygmund singular integrals (see [15]). In [3], the authors considered the series (1.2) with the Poisson operator related with translation semigroups \(f(t-s)\).

In order to analyze the series

$$\begin{aligned} \sum _{j\in {\mathbb Z}} v_j({\mathcal P}^\alpha _{a_{j+1}} f(x)-{\mathcal P}^\alpha _{a_j} f(x)), \end{aligned}$$

where \({\mathcal P}_t^\alpha \) is the generalized Poisson operator defined in (1.1), we can consider the convergence of its partial sums. For each \(N\in {\mathbb Z}^2,~N=(N_1,N_2)\) with \(N_1<N_2,\) we define the sum

$$\begin{aligned} T_N^\alpha f(x)&=\sum _{j=N_1}^{N_2} v_j({\mathcal P}_{a_{j+1}}^\alpha f(x)-{\mathcal P}_{a_j}^\alpha f(x)) \nonumber \\&= \frac{1}{4^\alpha \Gamma (\alpha )} \sum _{j=N_1}^{N_2}v_j \int _{{\mathbb R}^n}\int _0^\infty \frac{ a_{j+1}^{2\alpha }e^{-a_{j+1}^2/(4 s)}-a_j^{2\alpha } e^{-a_j^2/(4 s)} }{s^{1+\alpha }} \frac{e^{-|y|^2/(4s)}}{(4\pi s)^{n/2}} f(x-y)~{\textrm{d}}s {\textrm{d}}y \nonumber \\&= \frac{1}{4^\alpha \Gamma (\alpha )} \int _{{\mathbb R}^{n}} \int _0^{+\infty }\sum _{j=N_1}^{N_2}v_j \frac{ a_{j+1}^{2\alpha }e^{-a_{j+1}^2/(4 s)}-a_j^{2\alpha } e^{-a_j^2/(4 s)} }{s^{1+\alpha }} \frac{e^{-|y|^2/(4s)}}{(4\pi s)^{n/2}}~{\textrm{d}}s f(x-y) {\textrm{d}}y. \end{aligned}$$
(1.3)

We denote the kernel of \(T_N^\alpha \) by

$$\begin{aligned}K_N^\alpha (y)=\frac{1}{4^\alpha \Gamma (\alpha )}\int _0^{+\infty } \sum _{j=N_1}^{N_2}v_j \frac{ a_{j+1}^{2\alpha }e^{-a_{j+1}^2/(4 s)}-a_j^{2\alpha } e^{-a_j^2/(4 s)}}{s^{1+\alpha }} \frac{e^{-|y|^2/(4s)}}{(4\pi s)^{n/2}}{\textrm{d}}s.\end{aligned}$$

We shall also consider the maximal operators

$$\begin{aligned} T^*f(x)=\sup _N \left| T^\alpha _N f(x)\right| , \quad x\in {\mathbb R}^n, \end{aligned}$$

where the supremum are taken over all \(N=(N_1,N_2)\in {\mathbb Z}^2\) with \(N_1< N_2\). We shall consider the boundedness problem related to these operators. In [3], the authors proved the boundedness of the above operators related with the one-sided generalized Poisson type operator sequence.

Some of our results will be valid only when the sequence \(\{a_j\}_{j\in \mathbb Z}\) is lacunary. It means that there exists a \(\rho >1\) such that \(\displaystyle \frac{a_{j+1}}{a_j} \ge \rho , \, j \in \mathbb {Z}\). In particular, we shall prove the boundedness of the operators \(T^*\) in the weighted spaces \(L^p(\mathbb R^{n}, \omega ),\) where \(\omega \) is the usual Muckenhoupt weights on \(\mathbb R^{n}\). We refer the reader to the book by J. Duoandikoetxea [7, Chapter 7] for definitions and properties of the \(A_p\) classes. We have the following results:

Theorem 1

  1. (a)

    For any \(1<p<\infty \) and \(\omega \in A_p\), there exists a constant C depending on \(n, p, \rho , \alpha \) and \(\left\| v\right\| _{l^\infty (\mathbb Z)}\) such that

    $$\begin{aligned}\left\| T^*f\right\| _{L^p(\mathbb R^n, \omega )}\le C\left\| f\right\| _{L^p(\mathbb R^n, \omega )},\end{aligned}$$

    for all functions \(f\in L^p(\mathbb R^n, \omega ).\)

  2. (b)

    For any \(\omega \in A_1\), there exists a constant C depending on \(n, \rho , \alpha \) and \(\left\| v\right\| _{l^\infty (\mathbb Z)}\) such that

    $$\begin{aligned}\omega \left( {\{x\in {\mathbb R}^n:\left| T^*f(x)\right|>\lambda \}}\right) \le C\frac{1}{\lambda }\left\| f\right\| _{L^1(\mathbb R^n, \omega )}, \quad \lambda >0,\end{aligned}$$

    for all functions \(f\in L^1(\mathbb R^n, \omega ).\)

  3. (c)

    Given \(f\in L^\infty ({\mathbb R}^n),\) then either \(T^* f(x) =\infty \) for all \(x\in \mathbb R^n\), or \(T^* f(x) < \infty \) for a.e. \(x\in \mathbb R^n\). And in this latter case, there exists a constant C depending on \(n, \rho \), \(\alpha \) and \(\left\| v\right\| _{l^\infty (\mathbb Z)}\) such that

    $$\begin{aligned}\left\| T^*f\right\| _{BMO(\mathbb R^n)}\le C\left\| f\right\| _{L^\infty (\mathbb R^n)}.\end{aligned}$$
  4. (d)

    Given \(f\in BMO({\mathbb R}^n),\) then either \(T^* f(x) =\infty \) for all \(x\in \mathbb R^n\), or \(T^* f(x) < \infty \) for a.e. \(x\in \mathbb R^n\). And in this latter case, there exists a constant C depending on \(n, \rho \), \(\alpha \) and \(\left\| v\right\| _{l^\infty (\mathbb Z)}\) such that

    $$\begin{aligned} \left\| T^*f\right\| _{ BMO(\mathbb R^n)}\le C\left\| f\right\| _{BMO(\mathbb R^n)}. \end{aligned}$$
    (1.4)

Remark 1

From the conclusions we got in Theorem 1, for \(f\in L^p({\mathbb R}^{n}, \omega )\) with \(\omega \in A_p\), in Theorem 8 we shall see that we can define Tf by the limit of \(T_N f\) in \(L^p\)-norm

$$\begin{aligned}T f(x)=\lim _{(N_1,N_2)\rightarrow (-\infty , +\infty )} T_N f(x),\quad \quad ~x\in {\mathbb R}^{n}.\end{aligned}$$

In classical harmonic analysis, if \(f= \chi _{(0,1)}\) and \(\mathcal {H}\) is the Hilbert transform, it is easy to see that \(\displaystyle \frac{1}{r} \int _{-r}^0 \mathcal {H}(f)(x){\textrm{d}}x\ \sim \log \frac{e}{r}\) as \( r \rightarrow 0^+\). In general, this is the growth of a singular integral applied to a bounded function at the origin. The following theorem shows that the growth of the function \(T^*f\) for bounded function f at the origin is of the same order of a singular integral operator. Some related results about the local behavior of variation operators can be found in [1]. One-dimensional results about the variation of some convolutions operators can be found in [14]. And the one-dimensional results about the differential transforms of one-sided fractional Poisson type operator sequence is proved in [3]. In [4], the authors got local growth of the differential transforms of heat semigroup generated by Laplacian.

Theorem 2

  1. (a)

    Let \(\{v_j\}_{j\in \mathbb Z}\in l^p(\mathbb Z)\) for some \(1 \le p\le \infty .\) For every \(f\in L^\infty (\mathbb {R}^n)\) with support in the unit ball \(B=B(0, 1)\), for any ball \(B_r\subset B\) with \(2r<1\), we have

    $$\begin{aligned}\frac{1}{|B_r|} \int _{B_r} \left| T^* f (x)\right| {\textrm{d}}x\le C\left( \log \frac{2}{r}\right) ^{1/p'}\left\| v\right\| _{l^p(\mathbb Z)}\Vert f\Vert _{L^\infty (\mathbb R^n)}.\end{aligned}$$
  2. (b)

    When \(1< p<\infty \), for any \(0<\varepsilon <p-1\), there exist a \(\rho \)-lacunary sequence \(\{a_j\}_{j\in \mathbb Z}\), a sequence \(\{v_j\}_{j\in \mathbb Z}\in \ell ^p(\mathbb Z)\) and a function \(f\in L^\infty (\mathbb {R}^n)\) with support in the unit ball \(B=B(0, 1),\) satisfying the following statement: for any ball \(B_r\subset B\) with \(2r<1\), we have

    $$\begin{aligned}\frac{1}{|B_r|} \int _{B_r} \left| T^* f (x)\right| {\textrm{d}}x\ge C\left( \log \frac{2}{r}\right) ^{1/(p-\varepsilon )'}\left\| v\right\| _{l^p(\mathbb Z)}\Vert f\Vert _{L^\infty (\mathbb R^n)}.\end{aligned}$$
  3. (c)

    When \(p=\infty ,\) there exist a \(\rho \)-lacunary sequence \(\{a_j\}_{j\in \mathbb Z}\), a sequence \(\{v_j\}_{j\in \mathbb Z}\in l^\infty (\mathbb Z)\) and \(f\in L^\infty (\mathbb {R}^n)\) with support in the unit ball \(B=B(0, 1)\), satisfying the following statements: for any ball \(B_r\subset B\) with \(2r<1\),

    $$\begin{aligned}\frac{1}{|B_r|} \int _{B_r} \left| T^* f (x)\right| {\textrm{d}}x\ge C\left( \log \frac{2}{r}\right) \left\| v\right\| _{l^\infty (\mathbb Z)}\Vert f\Vert _{L^\infty (\mathbb R^n)}.\end{aligned}$$

In the statements above, \(\displaystyle p' = \frac{p}{p-1},\) and if \(p=1\), \(\displaystyle p'=\infty .\)

The statements in Theorem 2 shows that, when \(1< p<\infty ,\) the growth of \(T^*\) is between the growth of the standard singular integral and the growth of the Hardy–Littlewood maximal operator. And when \(p=\infty ,\) the growth of \(T^*\) is the same with the standard singular integral operator.

The organization of the paper is as follows: Sect. 2 is devoted to prove the boundedness of the maximal operators \(T^*\). And we will give the proof of the local growth of \(T^*\), i.e. Theorem 2, in Sect. 3. In Sect. 4, we will get some similar results in the Schrödinger setting.

Throughout this paper, the symbol C in an inequality always denotes a constant which may depend on some indices, but never on the functions f under consideration.

2 Proof of Theorem 1

In this section, we will prove Theorem 1. In order to prove Theorem 1, we need to prove the uniform boundedness of \(T_N^\alpha \) first. By the Fourier transform, we can prove that the operators \(T_N^\alpha \) are uniform bounded in \(L^2({\mathbb R}^{n})\) for all \(N\in {\mathbb Z}^2,~N_1< N_2\). Since the kernel \(K_N^\alpha (y,s)\) satisfies the size and smoothness conditions (see Theorem 5), we can deduce the \(L^p\)-boundedness results by using the Calderón–Zygmund theorem. Thus, we have the following results:

Theorem 3

For the operator \(T_N^\alpha \) defined in (1.3), we have the following statements.

  1. (a)

    For any \(1<p<\infty \) and \(\omega \in A_p\), there exists a constant C depending on \(n, p, \alpha \) and \(\left\| v\right\| _{l^\infty (\mathbb Z)}\) such that

    $$\begin{aligned}\left\| T_N^\alpha f\right\| _{L^p(\mathbb R^{n}, \omega )}\le C\left\| f\right\| _{L^p(\mathbb R^{n}, \omega )},\end{aligned}$$

    for all functions \(f\in L^p({\mathbb R}^{n}, \omega ).\)

  2. (b)

    For any \(\omega \in A_1\), there exists a constant C depending on \(n, \alpha \) and \(\left\| v\right\| _{l^\infty (\mathbb Z)}\) such that

    $$\begin{aligned}\omega \left( {\{x\in {\mathbb R}^{n}:\left| T_N^\alpha f(x)\right|>\lambda \}}\right) \le C\frac{1}{\lambda }\left\| f\right\| _{L^1(\mathbb R^{n}, \omega )},\quad \lambda >0,\end{aligned}$$

    for all functions \(f\in L^1({\mathbb R}^{n}, \omega ).\)

The constants C appeared above all are independent of N.

We shall use the Calderón–Zygmund theory in proving the \(L^p\)-boundedness of the differential transforms \(T_N^\alpha \) associated with the generalized Poisson operators. We will prove the \(L^2\)-estimates first. And then, it remains to give the estimates about the kernels of the differential transforms. By a standard argument, the results in Theorem 3 will be obtained.

First, we present a lemma which will be used later.

Lemma 1

( [3, Lemma 2.1]) Let \(0<\alpha <1\). Then for any complex number \(z_0\) with \(Re z_0 > 0\) and \(\displaystyle |\arg z_0 |\le {\pi }/{4}\), we have

$$\begin{aligned} \int _0^{+\infty } e^{-z_0 u} e^{-\frac{z_0}{u} }\,\frac{{\textrm{d}}u}{u^{\alpha }}= z_0^{1-\alpha }\int _{0}^{+\infty } \frac{e^{-r}e^{- z_0^2/r}}{r^{2-\alpha }} {\textrm{d}}r.\end{aligned}$$

Now we present the uniform \(L^2\)-boundedness of the operator \(T^\alpha _N\) in the following theorem:

Theorem 4

There exists a constant \(C>0\), depending on \(n, \alpha \) and \(\left\| v\right\| _{l^\infty (\mathbb Z)}\), such that

$$\begin{aligned}\sup _N \Vert T_N^\alpha f \Vert _{L^2({\mathbb R}^{n})}\le C \Vert f \Vert _{L^2({\mathbb R}^{n})}.\end{aligned}$$

Proof

Let \(f\in L^2({\mathbb R}^{n})\). Using the Plancherel theorem, we have

$$\begin{aligned} \left\| T_N^\alpha f \right\| _{L^2({\mathbb R}^{n})}&= \left\| \sum _{j=N_1}^{N_2} v_j({\mathcal P}_{a_{j+1}}^\alpha f -{\mathcal P}_{a_j}^\alpha f)\right\| _{L^2({\mathbb R}^{n})} \le C\left\| v\right\| _{l^\infty (\mathbb Z)} \left\| \sum _{j=-\infty }^{\infty } \int _{a_j}^{a_{j+1}} \left| \partial _t \widehat{{\mathcal P}_{t}^\alpha f }\right| {\textrm{d}}t\right\| _{L^2({\mathbb R}^{n})}. \end{aligned}$$

By using the second identity in (1.1), we have

$$\begin{aligned} \partial _t \widehat{{\mathcal P}_{t}^\alpha f }(\xi )&= C \partial _t \int _0^\infty e^{-r} \widehat{ e^{-\frac{t^2}{4r} \Delta } f}(\xi )\,\frac{{\textrm{d}}r}{r^{1-\alpha }}\\&= C \partial _t \int _0^\infty e^{-r} e^{-\frac{t^2}{4r}|\xi |^2}{\widehat{f}} (\xi ) \,\frac{{\textrm{d}}r}{r^{1-\alpha }}\\&= C \int _0^\infty e^{-r}t |\xi |^2 e^{-\frac{t^2}{4r}|\xi |^2}{\widehat{f}} (\xi ) \,\frac{{\textrm{d}}r}{r^{2-\alpha }}. \end{aligned}$$

Note that the Fourier transform above can be well defined. Then we deduce that

$$\begin{aligned} \left\| T_N^\alpha f \right\| _{L^2({\mathbb R}^{n})}&\le C \left\| {\widehat{f}} (\xi ) \int _{0}^\infty \left| \int _0^\infty e^{-r}t |\xi |^2 e^{-\frac{t^2}{4r}|\xi |^2}\,\frac{{\textrm{d}}r}{r^{2-\alpha }} \right| {\textrm{d}}t\right\| _{L^2({\mathbb R}^{n})}. \end{aligned}$$

Thus again by the Plancherel theorem, the remainder is devoted to prove the uniform boundedness of the multiplier

$$\begin{aligned}\left| \widehat{K_N^\alpha }(\xi )\right| \le C \left| \int _{0}^\infty \left| \int _0^\infty e^{-r}t |\xi |^2 e^{-\frac{t^2}{4r}|\xi |^2}\,\frac{{\textrm{d}}r}{r^{2-\alpha }}\right| {\textrm{d}}t\right| \le C, \quad \xi \in {\mathbb R}^{n}.\end{aligned}$$

Taking \(z_0=t|\xi |\), we rewrite the above inequality in

$$\begin{aligned}\left| \widehat{K_N^\alpha }(\xi )\right| \le C\int _{0}^\infty \left| \int _0^\infty e^{-r}z_0 e^{-\frac{z_0^2}{4r} } \,\frac{{\textrm{d}}r}{r^{2-\alpha }} \right| {\textrm{d}}z_0, \quad \xi \in {\mathbb R}^{n}.\end{aligned}$$

By Lemma 1, for any \(\xi \in {\mathbb R}^{n}\), we have

$$\begin{aligned} \int _{0}^\infty \left| \int _0^\infty e^{-r}z_0 e^{-\frac{z_0^2}{4r} }\,\frac{{\textrm{d}}r}{r^{2-\alpha }} \right| {\textrm{d}}z_0 = 2^{1-\alpha }\int _{0}^\infty \left| z_0^\alpha \int _0^\infty e^{-\frac{z_0}{2u}}e^{-\frac{z_0}{2}u} \frac{{\textrm{d}}u}{u^\alpha } \right| {\textrm{d}}z_0.\end{aligned}$$

Since \(\left| \arg z_0\right| \le {\pi }/{4}\), we have \(|e^{-z_0/(2u)}| \le e^{-c|z_0|/u} \) and \(|e^{-z_0 u/2}| \le e^{-c|z_0| u}\), where \(c={ \sqrt{2}/ {4}}\). Then

$$\begin{aligned} \left| \int _{0}^\infty z_0^\alpha \int _0^\infty e^{-z_0/u}e^{-z_0 u} \frac{{\textrm{d}}u}{u^\alpha } {\textrm{d}}z_0\right|&\le \int _{0}^\infty \, |z_0|^\alpha \int _0^\infty e^{-c|z_0|/u}e^{-c|z_0| u} \frac{{\textrm{d}}u}{u^\alpha } {\textrm{d}}z_0\\&\le \int _{0}^\infty \,|z_0|^{2\alpha -1}\int _0^\infty e^{-c|z_0|^2/v}e^{-c v} \frac{{\textrm{d}}v}{v^\alpha } {\textrm{d}}z_0. \end{aligned}$$

Recall that \(z_0=t |\xi |\). Then, we have

$$\begin{aligned}&\int _{0}^\infty \,|z_0|^{2\alpha -1}\int _0^\infty e^{-c|z_0|^2/v}e^{-c v} \frac{{\textrm{d}}v}{v^\alpha } {\textrm{d}}z_0\\&= \int _{0}^\infty |\xi |^{2\alpha }\,t^{2\alpha -1}\int _0^\infty e^{-c(|\xi |t)^2/v}e^{-c v} \frac{{\textrm{d}}v}{v^\alpha } {\textrm{d}}t\\&= \int _{0}^\infty \int _0^\infty (|\xi |t)^{2\alpha -1} e^{-c(|\xi |t)^2/v} d(|\xi |t) e^{-c v} \frac{{\textrm{d}}v}{v^\alpha }\\&= \int _{0}^\infty \int _0^\infty t^{2\alpha -1} e^{-ct^2/v} {\textrm{d}}t e^{-c v} \frac{d v}{v^\alpha } \le C \int _{0}^\infty e^{-c v} {\textrm{d}}v \le C, \end{aligned}$$

where the constants C appeared above all are independent of N. Then the proof of the theorem is complete. \(\square \)

Also, we can get the kernel estimates in the following:

Theorem 5

There exists constant \(C>0\) depending on \(n, \alpha \) and \(\left\| v\right\| _{l^\infty (\mathbb Z)}\)(not on N) such that, for any \(y\ne 0,\)

  1. (i)

    \(\displaystyle |K_N^\alpha (y)|\le \frac{C}{|y|^{n}}\),

  2. (ii)

    \(\displaystyle |\nabla _y K_N^\alpha (y)| \le \frac{C}{|y|^{n+1}}\).

Proof

i) This is the size condition of the kernel. We have

$$\begin{aligned}{} & {} |K_N^\alpha (y)| \le C\int _0^{+\infty }\sum _{j=-\infty }^{\infty } \left| \frac{a_{j+1}^{2\alpha } e^{-a_{j+1}^2/(4s)}-a_j^{2\alpha } e^{-a_j^2/(4s)}}{s^{1+\alpha }} \frac{e^{-|y|^2/(4s)}}{s^{n/2}} \right| {\textrm{d}}s\\{} & {} \quad = C\int _0^{+\infty }\sum _{j=-\infty }^{\infty } \left| a_{j+1}^{2\alpha } e^{-a_{j+1}^2/(4s)}-a_j^{2\alpha } e^{-a_j^2/(4s)}\right| \frac{e^{-|y|^2/(4s)}}{s^{1+\alpha +n/2}}{\textrm{d}}s. \end{aligned}$$

Observe that

$$\begin{aligned}&\sum _{j=-\infty }^{\infty } \left| a_{j+1}^{2\alpha }e^{-a_{j+1}^2/(4s)}-a_j^{2\alpha }e^{-a_j^2/(4s)}\right| = \sum _{j=-\infty }^{\infty } \left| \int _{a_j}^{a_{j+1}}\partial _u \left( u^{2\alpha } e^{-u^2/(4s)}\right) {\textrm{d}}u\right| \nonumber \\&\quad \le \int _{0}^{+\infty }\left| ( 2\alpha u^{2\alpha -1}-\frac{u^{2\alpha +1}}{2s})e^{-u^2/(4s)} \right| {\textrm{d}}u \le C\int _{0}^{+\infty }\left| (u^{2\alpha -1}+\frac{u^{2\alpha +1}}{2s})e^{-u^2/(4s)} \right| {\textrm{d}}u\nonumber \\&\quad \le C\sqrt{s}\Big (\int _{0}^{+\infty } (\sqrt{s} )^{2\alpha -1} \left( \frac{u}{\sqrt{s}} \right) ^{2\alpha -1} e^{-\frac{1}{4} \left( u/\sqrt{s}\right) ^2} d\frac{u}{\sqrt{s}}\nonumber \\&\qquad \,+ s^{\alpha -1/2}\int _{0}^{+\infty }\left( \frac{u}{\sqrt{s}}\right) ^{2\alpha +1} e^{-\frac{1}{4}\left( u/\sqrt{s} \right) ^2} d\frac{u}{\sqrt{s}}\Big )\nonumber \\&\quad \le C s^{\alpha }. \end{aligned}$$
(2.1)

Then, we have

$$\begin{aligned} |K_N^\alpha (y)| \le C \int _0^{+\infty }\frac{e^{-|y|^2/(4s)}}{s^{n/2+1}}{\textrm{d}}s=C \left( \int _0^{|y|^2}+\int _{|y|^2}^{+\infty }\right) \frac{e^{-|y|^2/(4s)}}{s^{n/2+1}}{\textrm{d}}s\\ \le C \left( \int _0^{|y|^2}\frac{1}{|y|^{n+2}}{\textrm{d}}s+\int _{|y|^2}^{+\infty }\frac{1}{s^{n/2+1}}{\textrm{d}}s\right) \le \frac{C}{|y|^{n}}. \end{aligned}$$

ii) It suffices to prove that for the first variable \(y_1\in {\mathbb R}\), we have

$$\begin{aligned} |\partial _{y_1} K_N^\alpha (y) | \le \frac{C}{|y|^{n+1}},\end{aligned}$$

where

$$\begin{aligned} \partial _{y_1} K_N^{\alpha }(y)&=-\int _0^{+\infty }\sum _{j=N_1}^{N_2} v_j \frac{a_{j+1}^{2\alpha }e^{-a_{j+1}^2/(4s)}-a_j^{2\alpha }e^{-a_j^2/(4s)}}{s^{1+\alpha }} \frac{y_1 e^{-|y|^2/(4s)}}{2 s^{n/2+1}}{\textrm{d}}s. \end{aligned}$$

Then, by (2.1) we conclude that

$$\begin{aligned}&|\partial _{y_1} K_N^\alpha (y)| \le C \int _0^{+\infty } s^{\alpha }~\frac{y_1 e^{-|y|^2/(4s)}}{ s^{2+\alpha +n/2} }{\textrm{d}}s\le C \int _0^{+\infty }\frac{\frac{|y|}{\sqrt{s}} e^{-|y|^2/(4s)}}{ s^{n/2+3/2}}{\textrm{d}}s\\&\quad \le C\int _0^{+\infty } \frac{ e^{-|y|^2/(8s)}}{ s^{n/2+3/2}}{\textrm{d}}s=C\left( \int _0^{|y|^2}+\int _{|y|^2}^{+\infty }\right) \frac{ e^{-|y|^2/(8s)}}{ s^{n/2+3/2}}{\textrm{d}}s \le \frac{C}{|y|^{n+1}}. \end{aligned}$$

The proof of the theorem is complete. \(\square \)

Remark 2

If we consider an \(l^\infty (\mathbb Z^2)\)-valued operator \(Q: f\mapsto \left\{ T_N^\alpha f(x)\right\} _{N\in \mathbb Z^2}\) on the homogeneous space \(({\mathbb R}^{n}, d, {\textrm{d}}x)\), then \(T^*_\alpha f(x)=\left\| Qf(x)\right\| _{l^\infty ({\mathbb R}^{n})}\), and by Theorem 5, we know that the kernel of the operator Q is an \(l^\infty (\mathbb Z^2)\)-valued Calderón–Zygmund kernel.

In the next result, we will take care of the behavior of \(T_N^\alpha \) on \(BMO({\mathbb R}^n)\).

Theorem 6

Let \(\{a_j\}_{j\in \mathbb Z}\) be an increasing sequence. There exists a constant C depending on \(n, \alpha \) and \(\left\| v\right\| _{\ell ^\infty (\mathbb Z)}\)(not on N) such that

$$\begin{aligned}\left\| T_N^\alpha f\right\| _{BMO({\mathbb R}^n)}\le C\left\| f\right\| _{L^\infty ({\mathbb R}^n)},\end{aligned}$$

and

$$\begin{aligned}\left\| T_N^\alpha f\right\| _{BMO({\mathbb R}^n)}\le C\left\| f\right\| _{BMO({\mathbb R}^n)}.\end{aligned}$$

Proof

The finiteness of \(T_N^\alpha \) for functions in \(L^\infty ({\mathbb R}^n)\) is obvious, since for each N, \(K_N^\alpha \) is an integrable function. On the other hand, assume that \(f\in BMO({\mathbb R}^n)\). Let \(B=B(x_0, r_0)\) and \(B^*=B(x_0, 2r_0)\) with \(x_0\in {\mathbb R}^n\) and \(r_0>0\). We decompose f to be

$$\begin{aligned}f=(f-f_B)\chi _{B^*}+(f-f_B)\chi _{(B^*)^c}+f_B=f_1+f_2+f_3.\end{aligned}$$

By Theorem 4, we have

$$\begin{aligned} \int _{{\mathbb R}^n}\left| T_N^\alpha f_1\right| ^2{\textrm{d}}x\le C \left\| f_1\right\| ^2_{L^2({\mathbb R}^n)}\le C\int _{B^*}\left| f(x)-f_B\right| ^2{\textrm{d}}x\le C|B|\left\| f\right\| _{BMO({\mathbb R}^n)}^2.\nonumber \\ \end{aligned}$$
(2.2)

This means that \(T_N^\alpha f_1(x)<\infty ,\) \(a.e.\ x\in {\mathbb R}^n.\) And we should note that \(T_N^\alpha f_3(x)\equiv 0\), since \({\mathcal P}_{a_j}^\alpha f_3\equiv f_B\) for any \(j\in \mathbb Z.\) For \(T_N^\alpha f_2\), we note that, for any \(x\in B\) and \(t>0\),

$$\begin{aligned} {\mathcal P}_t^\alpha f_2(x)&=\frac{t^{2\alpha }}{4^\alpha \Gamma (\alpha )} \int _0^{+\infty } \int _{{\mathbb R}^n} \frac{e^{-(t^2+|x-y|^2)/(4 s)}}{(4\pi s)^{n/2}} f_2 (y){\textrm{d}}y \frac{{\textrm{d}}s}{s^{1+\alpha }}\\&\le C\frac{t^{2\alpha }}{4^\alpha \Gamma (\alpha )} \int _0^{+\infty } \sum _{k=1}^{+\infty }\int _{2^kr_0<|x_0-y|\le 2^{k+1}r_0} {1\over |x-y|^{n+2\alpha '}}\left| f(y)-f_B\right| {\textrm{d}}y\ {e^{-{t^2\over 4 s}}}\frac{{\textrm{d}}s}{s^{1+\alpha -\alpha '}}\\&\le C\frac{t^{2\alpha '}\Gamma (\alpha -\alpha ')}{4^{\alpha '}\Gamma (\alpha )} \sum _{k=1}^{+\infty }{(2^kr_0)}^{-2\alpha '}{1\over {(2^kr_0)}^{n}}\int _{|x_0-y|\le 2^{k+1}r_0} \left| f(y)-f_B\right| {\textrm{d}}y\\&\le C\frac{t^{2\alpha '}\Gamma (\alpha -\alpha ')}{4^{\alpha '}\Gamma (\alpha )} \sum _{k=1}^{+\infty }{(2^kr_0)}^{-2\alpha '}(1+2k)\left\| f\right\| _{BMO({\mathbb R}^n)}<\infty , \end{aligned}$$

where \(0<\alpha '<\alpha .\) So, \({\mathcal P}_t^\alpha f_2(x)\) is well defined for \(x\in B\) and \(t>0.\) Since \(T_N^\alpha f_2(x)\) is a finite summation and \(x_0, r_0\) is arbitrary, \(T_N^\alpha f_2(x)<\infty \) a.e. \(x\in {\mathbb R}^n.\) Hence, \(T_N^\alpha f(x)<\infty \) a.e. \(x\in {\mathbb R}^n.\)

Now, let us prove the two inequalities. Since \(L^\infty ({\mathbb R}^n)\subset BMO({\mathbb R}^n),\) we only need to prove the inequality in the case of \(f\in BMO({\mathbb R}^n).\) Choose some \(x_1\in B(x_0, r_0)\) such that \(T_N^\alpha f_2(x_1)<\infty .\) Now, taking a constant \(c_B=T_N^\alpha f_2(x_1)\), we can write

$$\begin{aligned}&{1\over |B|}\int _B\left| T_N^\alpha f(x)-c_B\right| {\textrm{d}}x \\&= {1\over |B|}\int _B\left| T_N^\alpha (f_1+f_2+f_3)(x)-T_N^\alpha f_2(x_1)\right| {\textrm{d}}x\\&\le {1\over |B|}\int _B\left| T_N^\alpha f_1(x)\right| {\textrm{d}}x+{1\over |B|}\int _B\left| T_N^\alpha f_2(x)-T_N^\alpha f_2(x_1)\right| {\textrm{d}}x\\&=:I_1+I_2. \end{aligned}$$

For the first term \(I_1\), by Hölder’s inequality and (2.2) we have

$$\begin{aligned} I_1={1\over |B|}\int _B\left| T_N^\alpha f_1(x)\right| {\textrm{d}}x\le \left( {1\over |B|}\int _B\left| T_N^\alpha f_1(x)\right| ^2{\textrm{d}}x\right) ^{1/2}\le C\left\| f\right\| _{BMO({\mathbb R}^n)}. \end{aligned}$$

For the term \(I_2\), by Theorem 5ii) we have

$$\begin{aligned} I_2&={1\over |B|}\int _B\left| T_N^\alpha f_2(x)-T_N^\alpha f_2(x_1)\right| {\textrm{d}}x\\&={1\over |B|}\int _B\left| \int _{{\mathbb R}^n}\left( K_N^\alpha (x-y)-K_N^\alpha (x_1-y) \right) f_2(y){\textrm{d}}y\right| {\textrm{d}}x\\&\le {1\over |B|}\int _B\int _{{\mathbb R}^n}\left| K_N^\alpha (x-y)-K_N^\alpha (x_1-y)\right| \left| f_2(y)\right| {\textrm{d}}y{\textrm{d}}x\\&\le C{1\over |B|}\int _B\int _{(B^*)^c}{\left| x-x_1\right| \over \left| x-y\right| ^{n+1}}\left| f(y)-f_B\right| {\textrm{d}}y{\textrm{d}}x\\&\le C{r_0\over |B|}\int _B\int _{(B^*)^c}{1\over (\left| y-x_0\right| -\left| x_0-x\right| )^{n+1}}\left| f(y)-f_B\right| {\textrm{d}}y{\textrm{d}}x\\&= C{r_0\over |B|} \int _B \sum _{k=1}^{+\infty }\int _{2^kr_0\le \left| y-x_0\right|<2^{k+1}r_0}{1\over (\left| y-x_0\right| -\left| x_0-x\right| )^{n+1}}\left| f(y)-f_B\right| {\textrm{d}}y{\textrm{d}}x\\&\le C\sum _{k=1}^{+\infty }{1\over 2^k}{1\over (2^kr_0)^n} \int _{ \left| y-x_0\right| <2^{k+1}r_0}\left| f(y)-f_B\right| {\textrm{d}}y\\&\le C\left\| f\right\| _{BMO({\mathbb R}^n)}. \end{aligned}$$

Hence, we deduce

$$\begin{aligned} {1\over |B|}\int _B\left| T_N^\alpha f(x)-c_B\right| {\textrm{d}}x\le C\left\| f\right\| _{BMO({\mathbb R}^n)}. \end{aligned}$$

Thus, we proved that \(T_N^\alpha f\in BMO({\mathbb R}^n)\) and

$$\begin{aligned}\left\| T_N^\alpha f\right\| _{BMO({\mathbb R}^n)}\le C\left\| f\right\| _{BMO({\mathbb R}^n)}.\end{aligned}$$

\(\square \)

In the following, we aim to prove Theorem 1. The next proposition, parallel to Proposition 3.2 in [2](also Proposition 3.1 in [3]), shows that, without lost of generality, we may assume that

$$\begin{aligned} 1<\rho \le {a_{j+1} \over a_j}\le \rho ^2, \quad j\in \mathbb Z. \end{aligned}$$

Proposition 1

Given a \(\rho \)-lacunary sequence \(\{a_j\}_{j\in \mathbb Z}\) and a multiplying sequence \(\{v_j\}_{j\in \mathbb Z}\in \ell ^\infty (\mathbb Z)\), we can define a \(\rho \)-lacunary sequence \(\{\eta _j\}_{j\in \mathbb Z}\) and \(\{\omega _j\}_{j\in \mathbb Z}\in \ell ^\infty (\mathbb Z)\) verifying the following properties:

  1. (i)

    \(1<\rho \le \eta _{j+1}/\eta _j\le \rho ^2,\quad \quad \left\| \{\omega _j\}\right\| _{\ell ^\infty (\mathbb Z)}=\left\| \{v_j\}\right\| _{\ell ^\infty (\mathbb Z)}\).

  2. (ii)

    For all \(N=(N_1, N_2)\) there exists \(N'=(N_1', N_2')\) with \(T^\alpha _N={\hat{T}}^\alpha _{N'},\) where \({\hat{T}}^\alpha _{N'}\) is the operator defined in (1.3) with the new sequences \(\{\eta _j\}_{j\in {\mathbb Z}}\) and \(\{\omega _j\}_{j\in {\mathbb Z}}.\)

In order to prove Theorem 1, we need a Cotlar’s type inequality. For any \(M\in \mathbb Z^+,\) let

$$\begin{aligned}T_M^*f(x)=\sup _{-M\le N_1<N_2\le M}\left| T_N f(x)\right| ,\quad x\in {\mathbb R}^n,\end{aligned}$$

where \(T_N\) denotes the differential transform operator related with the heat-diffusion semigroup generated by \(-\Delta \). By a similar(in fact, easier) argument as in the proof of Theorem 4, we can prove that \(T_N\) is uniform bounded on \(L^2({\mathbb R}^n)\). Also, we can prove that \(T_N\) is uniform bounded in \(L^p({\mathbb R}^n, \omega )\) for \(1<p<\infty \), uniform weak-(1, 1) bounded and uniform BMO-bounded, because it is a Calderón–Zygmund operator. For these results, see [4].

Theorem 7

(See [4, Theorem 2.4]) For each \(q\in (1, +\infty ),\) there exists a constant C depending on \(n, \left\| v\right\| _{l^\infty (\mathbb Z)}\) and \(\rho \) such that for every \(x\in {\mathbb R}^n\) and every \(M\in \mathbb Z^+\),

$$\begin{aligned} T_M^*f(x)\le C\left\{ {\mathcal M}(T_{-M, M} f)(x)+{\mathcal M}_q f(x)\right\} , \end{aligned}$$

where

$$\begin{aligned}T_{-M, M} f(x)={\sum _{j=-M}^{M} v_j(e^{a_{j+1}\Delta } f(x)-e^{a_j\Delta } f(x))}\end{aligned}$$

and

$$\begin{aligned}{\mathcal M}_qf(x)=\sup _{\varepsilon >0} \left( \frac{1}{|B(x, \varepsilon )|}\int _{B(x, \varepsilon )}\left| f(y)\right| ^q{\textrm{d}}y\right) ^{1\over q},\quad 1< q<\infty .\end{aligned}$$

Now, we are in a position to prove Theorem 1.

Proof of Theorem 1

For each \(\omega \in A_p,\) choose \(1<q<p<\infty \) such that \(\omega \in A_{p/q}.\) Then, it is well known that the maximal operators \({\mathcal M}\) and \( {\mathcal M}_q\) are bounded on \(L^p({\mathbb R}^n, \omega )\). On the other hand, since the operators \(T_N\) are uniformly bounded in \(L^p({\mathbb R}^n, \omega )\) with \(\omega \in A_p\). Hence, by Theorem 7, we have

$$\begin{aligned} \left\| T_M^*f\right\| _{L^p({\mathbb R}^n, \omega )}&\le C\left( \left\| {\mathcal M}(T_{-M, M} f)\right\| _{L^p({\mathbb R}^n, \omega )}+\left\| {\mathcal M}_q f\right\| _{L^p({\mathbb R}^n, \omega )}\right) \\ {}&\le C\left( \left\| T_{-M, M} f\right\| _{L^p({\mathbb R}^n, \omega )}+\left\| f\right\| _{L^p({\mathbb R}^n, \omega )}\right) \le C\left\| f\right\| _{L^p({\mathbb R}^n, \omega )}. \end{aligned}$$

Note that the constants C appeared above do not depend on M. Consequently, letting M increase to infinity, we get the proof of the \(L^p\) boundedness of the maximal operator \(T_\Delta ^*,\) where \(\displaystyle T_\Delta ^*f(x)=\sup _N \left| T_Nf(x)\right| .\)

We should note that

$$\begin{aligned} T^* f(x)&=\sup _{N\in \mathbb Z^2}\left| T_{N}^\alpha f(x)\right| =\sup _{N\in \mathbb Z^2}\left| \sum _{j=N_1}^{N_2} v_j({\mathcal P}_{a_{j+1}}^\alpha f(x)-{\mathcal P}_{a_j}^\alpha f(x))\right| \\&=C_\alpha \sup _{N\in \mathbb Z^2}\left| \sum _{j=N_1}^{N_2} v_j\left( \int _0^{+\infty } e^{-s} \left( e^{-\frac{a_{j+1}^2}{4s} \Delta } f(x)- e^{-\frac{a_j^2}{4s} \Delta } f(x)\right) \,\frac{d s}{s^{1-\alpha }}\right) \right| \\&\le C_\alpha \int _0^{+\infty } e^{-s} \sup _{N\in \mathbb Z^2}\left| \sum _{j=N_1}^{N_2} v_j\left( e^{-\frac{a_{j+1}^2}{4s} \Delta } f(x)- e^{-\frac{a_j^2}{4s} \Delta } f(x)\right) \right| \,\frac{d s}{s^{1-\alpha }}\\&=C_{\alpha , \rho , v, n} \int _0^{+\infty } e^{-s} {\bar{T}}_\Delta ^*f(x)\,\frac{d s}{s^{1-\alpha }}, \end{aligned}$$

where the operator \({\bar{T}}_{\Delta }^*\)(which is bounded on \(L^p(\mathbb R^n, \omega )\) and the boundedness constant is not depending on s) denotes the maximal differential transform related with \(\{v_j\}_{j\in \mathbb Z}\) and \(\rho ^2\)-lacunary sequence \(\left\{ {a_j^2/ 4s}\right\} _{j\in \mathbb Z}\). Then,

$$\begin{aligned} \left\| T^* f\right\| _{L^p({\mathbb R}^n, \omega )}\le C_{\alpha , \rho , v, n} \int _0^{+\infty } e^{-s} \left\| {\bar{T}}_\Delta ^*f\right\| _{L^p({\mathbb R}^n, \omega )}\,\frac{d s}{s^{1-\alpha }}\le C_{\alpha , \rho , v, n} \left\| f\right\| _{L^p({\mathbb R}^n, \omega )}. \end{aligned}$$

This completes the proof of part (a) of the theorem.

In order to prove (b), we consider the \(\ell ^\infty (\mathbb Z^2)\)-valued operator \(\mathcal {T}f(x) = \{ T_N^\alpha f(x) \}_{N\in \mathbb Z^2}\). Since \(\Vert \mathcal {T}f(x) \Vert _{\ell ^\infty (\mathbb Z^2)}= T^*f(x)\), by using (a) we know that the operator \(\mathcal {T}\) is bounded from \(L^p({\mathbb R}^n, \omega ) \) into \(L^p_{\ell ^\infty (\mathbb Z^2)}(\mathbb {R}^n, \omega ) \), for every \(1<p<\infty \) and \(\omega \in A_p\). The kernel of the operator \(\mathcal {T}\) is given by \(\mathcal {K}^\alpha (x) = \{ K^\alpha _N(x)\} _{N\in \mathbb Z^2}\). By Theorem 5 and the vector valued version of Theorem 7.12 in [7], we get that the operator \(\mathcal {T}\) is bounded from \(L^1(\mathbb {R}^n, \omega )\) into weak- \(L^1_{\ell ^\infty (\mathbb Z^2)}(\mathbb {R}^n, \omega )\) for \(\omega \in A_1\). Hence, as \(\Vert \mathcal {T}f(x) \Vert _{\ell ^\infty (\mathbb Z^2)}= T^*f(x)\), we get the proof of (b).

For (c),  we shall prove that if \(f\in L^\infty (\mathbb R^n)\) and there exists \(x_0\in \mathbb R^n\) such that \(T^*f(x_0)<\infty ,\) then \(T^*f(x)<\infty \) for a.e. \(x\in {\mathbb R}^n.\) Given \(x\ne x_0.\) Set \(f_1=f\chi _{B{(x_0, 4|x-x_0|)}}\) and \(f_2 = f-f_1\). Note that \(T^*\) is \(L^p\)-bounded for any \(1<p<\infty .\) Then \(T^*f_1(x)<\infty \), because \(f_1\in L^p(\mathbb R^n)\) for any \(1<p<\infty .\) On the other hand, by Theorem 5 we have

$$\begin{aligned}&\Big |T_N^\alpha f_2(x)-T_N^\alpha f_2(x_0)\Big |\\ {}&=\Big |\int _{{\mathbb R}^n} K_N^\alpha (x, y)f_2(y){\textrm{d}}y-\int _{{\mathbb R}^n} K_N^\alpha (x_0, y)f_2(y){\textrm{d}}y \Big |\\&=\Big | \int _{B^c(x_0, 4|x-x_0|)}\left( K_N^\alpha (x, y) - K_N^\alpha (x_0, y)\right) f(y){\textrm{d}}y\Big |\\&\le C \int _{B^c(x_0, 4|x-x_0|)} \frac{\left| x-x_0\right| }{\left| y-x_0\right| ^{n+1}}\left| f(y)\right| {\textrm{d}}y \\&\le C\left\| f\right\| _{L^\infty (\mathbb R)}< +\infty . \end{aligned}$$

Hence

$$\begin{aligned} \left\| T_N^\alpha f_2(x)-T_N^\alpha f_2(x_0)\right\| _{l^\infty (\mathbb Z^2)} \le C\left\| f\right\| _{L^\infty (\mathbb R^n)} \end{aligned}$$

and therefore \( T^*f(x) = \left\| T_N^\alpha f(x)\right\| _{l^\infty (\mathbb Z^2)} \le C < \infty .\) For the \(L^\infty -BMO\) boundedness, we will prove it later.

(d) Let \(x_0\in {\mathbb R}^n\) be one point such that \(T^*f(x_0)<\infty .\) Set \(B=B(x_0, 4\left| x-x_0\right| )\) with \(x\ne x_0\). And we decompose f to be

$$\begin{aligned} f=(f-f_B)\chi _B+(f-f_B)\chi _{B^c}+f_B=:f_1+f_2+f_3. \end{aligned}$$

Note that \(T^*\) is \(L^p\)-bounded for any \(1<p<\infty .\) Then \(T^*f_1(x)<\infty \), because \(f_1\in L^p(\mathbb R^n)\), for any \(1<p<\infty .\) And \(T^* f_3=0\), since \({\mathcal P}_{a_j}^\alpha f_3=f_3\) for any \(j\in \mathbb Z.\) On the other hand, by Theorem 5 we have

$$\begin{aligned}&\Big |T_N^\alpha f_2(x)-T_N^\alpha f_2(x_0)\Big |\\ {}&=\Big |\int _{{\mathbb R}^n} K_N^\alpha (x, y)f_2(y){\textrm{d}}y-\int _{{\mathbb R}^n} K_N^\alpha (x_0, y)f_2(y){\textrm{d}}y \Big |\\&=\Big | \int _{B^c}\left( K_N^\alpha (x, y) - K_N^\alpha (x_0, y)\right) f_2(y){\textrm{d}}y\Big |\\&\le C \int _{B^c} \frac{\left| x-x_0\right| }{\left| y-x_0\right| ^{n+1}}\left| f(y)-f_B\right| {\textrm{d}}y \\&\le C \sum _{k=1}^{+\infty }{ |x-x_0|} \int _{2^{k} B\setminus 2^{k-1}B} {\left| f(y)-f_B\right| \over |y-x_0|^{n+1}}{\textrm{d}}y\\&\le C \sum _{k=1}^{+\infty }{ |x-x_0|\over (2^{k+1}|x-x_0|)^{n+1}} \int _{2^kB} {\left| f(y)-f_B\right| }{\textrm{d}}y\\&\le C \sum _{k=1}^{+\infty }2^{-(k+1)}{ 1\over \left| 2^{k}B\right| } \int _{2^{k}B} \left( {\left| f(y)-f_{2^kB}\right| }+\sum _{l=1}^{k}\left| f_{2^lB}-f_{2^{l-1}B}\right| \right) {\textrm{d}}y\\&\le C\sum _{k=1}^{+\infty }2^{-(k+1)}{ 1\over \left| 2^{k}B\right| } \int _{2^{k}B} \left( {\left| f(y)-f_{2^kB}\right| }+2k\left\| f\right\| _{BMO({\mathbb R}^n)}\right) {\textrm{d}}y\\&\le C\sum _{k=1}^{+\infty }2^{-(k+1)}{(1+2k)\left\| f\right\| _{BMO({\mathbb R}^n)}}\\&\le C\left\| f\right\| _{BMO({\mathbb R}^n)}, \end{aligned}$$

where \(2^kB=B(x_0, 2^{k}\cdot 4|x-x_0|)\) for any \(k\in \mathbb N.\) Hence

$$\begin{aligned} \left\| T_N^\alpha f_2(x)-T_N^\alpha f_2(x_0)\right\| _{l^\infty (\mathbb Z^2)} \le C\left\| f\right\| _{BMO(\mathbb R^n)} \end{aligned}$$

and therefore \( T^*f(x) = \left\| T_N^\alpha f(x)\right\| _{l^\infty (\mathbb Z^2)} \le C < \infty .\)

Now, we shall prove the estimate (1.4) for functions such that \(T^*f(x) < \infty \, \, a.e.\) For any \(r>0\) and \(x_0\) such that \(T^*f(x_0) < \infty \), consider the ball \(B=B(x_0, r)\) and \(\displaystyle f_B={1\over |B|}\int _B f(x){\textrm{d}}x.\) Let

$$\begin{aligned}f=(f-f_B)\chi _{2B}+(f-f_B)\chi _{(2B)^c}+f_B=:f_1+f_2+f_3.\end{aligned}$$

We have \(T^*f_3(x)=0.\) Then,

$$\begin{aligned}&{1\over |B|}\int _{B}\left| T^*f(x)-(T^*f)_B\right| {\textrm{d}}x={1\over |B|}\int _{B}\left| {1\over |B|}\int _{B}\left( T^*f(x)-T^*f(y)\right) {\textrm{d}}y\right| {\textrm{d}}x\\&\le {1\over |B|^2}\int _{B}\int _{B}\left| T^*f(x)-T^*f(y)\right| {\textrm{d}}y{\textrm{d}}x\\&={1\over |B|^2}\int _{B}\int _{B}\left| \left\| T^\alpha _Nf(x)\right\| _{l^\infty (\mathbb Z^2)}-\left\| T^\alpha _Nf(y)\right\| _{l^\infty (\mathbb Z^2)}\right| {\textrm{d}}y{\textrm{d}}x\\&\le {1\over |B|^2}\int _{B}\int _B{\left\| T^\alpha _Nf(x)-T^\alpha _Nf(y)\right\| _{l^\infty (\mathbb Z^2)}}{\textrm{d}}y{\textrm{d}}x\\&\le {1\over |B|^2}\int _{B}\int _{B}{\left\| T^\alpha _Nf_1(x)-T^\alpha _N f_1(y)\right\| _{l^\infty (\mathbb Z^2)}}{\textrm{d}}y{\textrm{d}}x\\&\quad + {1\over |B|^2}\int _{B}\int _{B}{\left\| T^\alpha _Nf_2(x)-T^\alpha _Nf_2(y)\right\| _{l^\infty (\mathbb Z^2)}}{\textrm{d}}y{\textrm{d}}x\\&=:I+II. \end{aligned}$$

The Hölder inequality and \(L^2\)-boundedness of \(T^*\) imply that

$$\begin{aligned} I&\le {1\over |B|}\int _{B}{\left\| T^\alpha _Nf_1(x)\right\| _{l^\infty (\mathbb Z^2)}}{\textrm{d}}x +{1\over |B|}\int _{B}{\left\| T^\alpha _Nf_1(y)\right\| _{l^\infty (\mathbb Z^2)}}{\textrm{d}}y\\&\le \left( {1\over |B|}\int _{B}{\left\| T^\alpha _Nf_1(x)\right\| ^2_{l^\infty (\mathbb Z^2)}}{\textrm{d}}x\right) ^{1/2} +\left( {1\over |B|}\int _{B}{\left\| T^\alpha _Nf_1(y)\right\| ^2_{l^\infty (\mathbb Z^2)}}{\textrm{d}}y\right) ^{1/2}\\&\le C{1\over |B|^{1/2}}\left\| f_1\right\| _{L^2(\mathbb R^n)}\le C\left\| f\right\| _{BMO(\mathbb R^n)}. \end{aligned}$$

For II, since \(x, y \in B\) and the support of \(f_2\) is \((2B)^c\), by Theorem 5 we have

$$\begin{aligned}&\Big |T_N^\alpha f_2(x)-T_N^\alpha f_2(y)\Big |\\&=\Big |\int _{{\mathbb R}^n} K_N^\alpha (x, z)f_2(z){\textrm{d}}z-\int _{{\mathbb R}^n} K_N^\alpha (y, z)f_2(z){\textrm{d}}z \Big |\\&=\Big | \int _{(2B)^c}\left( K_N^\alpha (x, z) - K_N^\alpha (y, z)\right) f_2(z){\textrm{d}}z\Big |\\&\le C \sum _{k=2}^{+\infty }{ r} \int _{2^{k} B\setminus 2^{k-1}B} {\left| f(z)-f_B\right| \over |z-x_0|^{n+1}}{\textrm{d}}z\\&\le C \sum _{k=2}^{+\infty }{ r\over (2^{k}r)^{n+1}} \int _{2^kB} {\left| f(z)-f_B\right| }{\textrm{d}}z\\&\le C \sum _{k=2}^{+\infty }2^{-k}{ 1\over \left| 2^{k}B\right| } \int _{2^{k}B} \left( {\left| f(z)-f_{2^kB}\right| }+\sum _{l=1}^{k}\left| f_{2^lB}-f_{2^{l-1}B}\right| \right) {\textrm{d}}z\\&\le C\sum _{k=2}^{+\infty }2^{-k}{ 1\over \left| 2^{k}B\right| } \int _{2^{k}B} \left( {\left| f(z)-f_{2^kB}\right| }+2k\left\| f\right\| _{BMO({\mathbb R}^n)}\right) {\textrm{d}}z\\&\le C\sum _{k=2}^{+\infty }2^{-k}{(1+2k)\left\| f\right\| _{BMO({\mathbb R}^n)}}\\&\le C\left\| f\right\| _{BMO({\mathbb R}^n)}, \end{aligned}$$

where \(2^kB=B(x_0, 2^kr)\). Hence, we have \(II\le C \left\| f\right\| _{BMO({\mathbb R}^n)}.\) Then by the arbitrary of \(x_0\) and \(r>0\), we proved

$$\begin{aligned}\left\| T^*f\right\| _{BMO(\mathbb R^n)}\le C\left\| f\right\| _{BMO(\mathbb R^n)}.\end{aligned}$$

For the second part of (c), we can deduce it from the BMO-boundedness of \(T^*\) and the inclusion \(L^\infty ({\mathbb R}^n)\subset BMO({\mathbb R}^n).\) This completes the proof of Theorem 1. \(\square \)

From the conclusions we got in Theorem 1, we have the following result:

Theorem 8

  1. (a)

    If \(1<p<\infty \) and \(\omega \in A_p\), then \(T^\alpha _N f\) converges a.e. and in \(L^p(\mathbb R^n, \omega )\) norms for all \(f\in L^p(\mathbb R^n, \omega )\) as \(N=(N_1,N_2)\) tends to \((-\infty , +\infty ).\)

  2. (b)

    If \(p=1\) and \(\omega \in A_1\), then \(T^\alpha _N f\) converges a.e. and in measure for all \(f\in L^1(\mathbb R^n, \omega )\) as \(N=(N_1,N_2)\) tends to \((-\infty , +\infty ).\)

Proof

First, we shall see that if \(\varphi \) is a test function, then \(T_N^\alpha \varphi (x)\) converges for all \(x\in {\mathbb R}^n\). In order to prove this, it is enough to see that for any \(N=(L,M)\) with \(0<L<M\), the series

$$\begin{aligned} A= \sum _{j=L}^M v_j ( {\mathcal P}^\alpha _{a_{j+1}} \varphi (x) - {\mathcal P}_{a_j}^\alpha \varphi (x)) \hbox { and } B= \sum _{j=-M}^{-L} v_j ( {\mathcal P}^\alpha _{a_{j+1}} \varphi (x) - {\mathcal P}_{a_j}^\alpha \varphi (x)) \end{aligned}$$

converge to zero, when \(L, M\rightarrow +\infty \). For A, by the mean value theorem and the \(\rho \)-lacunarity of the sequence \(\{a_j\}_{j\in \mathbb Z}\) we have

$$\begin{aligned} |A|&=C_\alpha \left| \sum _{j=L}^{M} v_j\left( \int _0^{+\infty } e^{-s} \left( e^{\frac{a_{j+1}^2}{4s} \Delta } \varphi (x)- e^{\frac{a_j^2}{4s} \Delta } \varphi (x)\right) \,\frac{d s}{s^{1-\alpha }}\right) \right| \\&\le C_\alpha \int _0^{+\infty } e^{-s}\left| \sum _{j=L}^{M} v_j \left( e^{\frac{a_{j+1}^2}{4s} \Delta } \varphi (x)- e^{\frac{a_j^2}{4s} \Delta } \varphi (x)\right) \right| \,\frac{d s}{s^{1-\alpha }}\\&= C_{n,\alpha } \int _0^{+\infty } e^{-s}\left| \sum _{j=L}^{M} v_j \left( \int _{{\mathbb R}^n}\frac{s^{n/2}}{a_{j+1}^n} e^{-\frac{s|y|^2}{a_{j+1}^2} } \varphi (x-y){\textrm{d}}y-\int _{{\mathbb R}^n}\frac{s^{n/2}}{a_{j}^n} e^{-\frac{s|y|^2}{a_{j}^2} } \varphi (x-y){\textrm{d}}y \right) \right| \,\frac{d s}{s^{1-\alpha }}\\&\le C_{n, \alpha ,v,\rho } \int _0^{+\infty } e^{-s}\int _{{\mathbb R}^n}\sum _{j=L}^{M} \frac{s^{n/2}}{a_j^n}|\varphi (x-y|{\textrm{d}}y \,\frac{d s}{s^{1-\alpha }}\\&\le C_{n, \alpha ,v,\rho }\left( \frac{1}{a_L^{n}}\sum _{j=L}^{M} \frac{a_L^{n}}{a_j^n}\right) \int _0^{+\infty } e^{-s}\int _{{\mathbb R}^n}|\varphi (x-y|{\textrm{d}}y \,\frac{d s}{s^{1-\alpha -n/2}}\\&\le C_{n, \alpha ,v,\rho }\left\| \varphi \right\| _{L^1({\mathbb R}^n)}\frac{1}{a_L^n}\rightarrow 0, \quad \text {as}\, \, \, L,M\rightarrow +\infty . \end{aligned}$$

For B,  as the integral of the kernels are zero, we can write

$$\begin{aligned} B&=C_{n,\alpha } \int _{{\mathbb R}^{n+1}} \sum _{j=-M}^{-L}v_j \frac{ a_{j+1}^{2\alpha }e^{-a_{j+1}^2/(4 s)}-a_j^{2\alpha } e^{-a_j^2/(4 s)}}{s^{1+\alpha }} {e^{-{|y|^n/(4s)}}\over (4\pi s)^{n/2}}( \varphi (x-y)- \varphi (x)){\textrm{d}}y{\textrm{d}}s\\&= C_{n,\alpha } \int _0^1 \int _{{\mathbb R}^n}\sum _{j=-M}^{-L}v_j \frac{ a_{j+1}^{2\alpha }e^{-a_{j+1}^2/(4 s)}-a_j^{2\alpha } e^{-a_j^2/(4 s)}}{s^{1+\alpha }} {e^{-{|y|^n/(4s)}}\over (4\pi s)^{n/2}}( \varphi (x-y)- \varphi (x)){\textrm{d}}y{\textrm{d}}s\\&\quad +C_{n,\alpha } \int _1^\infty \int _{{\mathbb R}^n}\sum _{j=-M}^{-L}v_j \frac{ a_{j+1}^{2\alpha }e^{-a_{j+1}^2/(4 s)}-a_j^{2\alpha } e^{-a_j^2/(4 s)}}{s^{1+\alpha }} {e^{-{|y|^n/(4s)}}\over (4\pi s)^{n/2}}( \varphi (x-y)- \varphi (x)){\textrm{d}}y{\textrm{d}}s\\&=: B_1+B_2. \end{aligned}$$

Proceeding as in the case A, and by using the fact that \(\varphi \) is a test function, we have

$$\begin{aligned} |B_1|&= C_{n,\alpha } \Big | \int _0^1 \int _{{\mathbb R}^n} \sum _{j=-M}^{-L}v_j \frac{ a_{j+1}^{2\alpha }e^{-a_{j+1}^2/(4 s)}-a_j^{2\alpha } e^{-a_j^2/(4 s)}}{s^{1+\alpha }} {e^{-{|y|^2/(4s)}}\over (4\pi s)^{n/2}}( \varphi (x-y)- \varphi (x)){\textrm{d}}y{\textrm{d}}s\Big | \\&\le C_{n,\alpha } \Big | \int _0^1 \int _{{\mathbb R}^n} \sum _{j=-M}^{-L}v_j \frac{ a_{j+1}^{2\alpha }e^{-a_{j+1}^2/(4 s)}-a_j^{2\alpha } e^{-a_j^2/(4 s)}}{s^{1+\alpha }} {e^{-{|y|^2/(4s)}}\over (4\pi s)^{n/2}}\left\| \nabla \varphi \right\| _{L^\infty ({\mathbb R}^{n+1})}|y|{\textrm{d}}y{\textrm{d}}s\Big | \\&\le C_{n,\alpha } \left\| \nabla \varphi \right\| _{L^\infty ({\mathbb R}^{n})} \int _0^{1}\int _{{\mathbb R}^n} \sum _{j=-M}^{-L}v_j \frac{ a_{j+1}^{2\alpha }e^{-a_{j+1}^2/(4 s)}-a_j^{2\alpha } e^{-a_j^2/(4 s)}}{s^{\alpha +1/2}}{e^{-{|y|^2/(4s)}}\over (4\pi s)^{n/2}} {\textrm{d}}y{\textrm{d}}s \\&\le C_{n,\alpha ,v,\rho } \left\| \nabla \varphi \right\| _{L^\infty ({\mathbb R}^{n})} \int _0^1 \sum _{j=-M}^{-L} \frac{a_{j}^{2\alpha } e^{-a_{j}^2/(4 s)}}{s^{\alpha +1/2}} {\textrm{d}}s. \end{aligned}$$

If \(\displaystyle 0<\alpha \le {1\over 2},\) then, for any \(0<\varepsilon <2\alpha ,\) we have

$$\begin{aligned} |B_1|&\le C_{n,\alpha , v,\rho } \left\| \nabla \varphi \right\| _{L^\infty ({\mathbb R}^{n})} \sum _{j=-M}^{-L} {a_{j}^{2\alpha -\varepsilon }}\int _0^1{s^{\varepsilon /2-\alpha -1/2}} {\textrm{d}}s\\&\le C_{n,\alpha ,v,\rho ,\varepsilon }\left\| \nabla \varphi \right\| _{L^\infty ({\mathbb R}^{n})} a_{-L}^{2\alpha -\varepsilon } \sum _{j=-M}^{-L} \frac{a_{j}^{2\alpha -\varepsilon }}{a_{-L}^{2\alpha -\varepsilon }} \\&\le C_{n,\alpha ,v,\rho ,\varepsilon } \left\| \nabla \varphi \right\| _{L^\infty ({\mathbb R}^{n})}a_{-L}^{2\alpha -\varepsilon }\longrightarrow 0,\quad \hbox {as}\ { L,M \rightarrow +\infty }. \end{aligned}$$

If \(\displaystyle {1\over 2}<\alpha <1,\) then

$$\begin{aligned} |B_1|&\le C_{n,\alpha , v,\rho } \left\| \nabla \varphi \right\| _{L^\infty ({\mathbb R}^{n})} \int _0^1 \sum _{j=-M}^{-L} \frac{a_{j}^{2\alpha } e^{-a_{j}^2/(4 s)}}{s^{\alpha +1/2}} {\textrm{d}}s\\&\le C_{n,\alpha ,v,\rho }\left\| \nabla \varphi \right\| _{L^\infty ({\mathbb R}^{n})} a_{-L}^{2\alpha -1} \sum _{j=-M}^{-L} \frac{a_{j}^{2\alpha -1}}{a_{-L}^{2\alpha -1}} \int _0^1 \frac{1}{s^{\alpha }} {\textrm{d}}s\\&\le C_{n,\alpha ,v,\rho } \left\| \nabla \varphi \right\| _{L^\infty ({\mathbb R}^{n})}a_{-L}^{2\alpha -1}\longrightarrow 0,\quad \hbox {as}\ { L,M \rightarrow +\infty }. \end{aligned}$$

Therefore, we get

$$\begin{aligned}|B_1|\longrightarrow 0,\quad \hbox {as}\ { L,M \rightarrow +\infty }.\end{aligned}$$

On the other hand,

$$\begin{aligned} |B_2|&\le C_{n,\alpha ,\rho } \left\| v\right\| _{l^\infty (\mathbb Z)} \Vert \varphi \Vert _{L^\infty ({\mathbb R}^{n}) }\int _1^\infty \sum _{j=-M}^{-L}\frac{ a_{j}^{2\alpha }}{s^{1+\alpha }} {\textrm{d}}s\\&\le C_{n, \alpha , v }\Vert \varphi \Vert _{L^\infty ({\mathbb R}^{n}) } \sum _{j=-M}^{-L}{ a_j^{2\alpha }}\int _1^\infty {1\over s^{1+\alpha }} {\textrm{d}}s \\&\le C_{n,\alpha , v, \rho } \Vert \varphi \Vert _{L^\infty ({\mathbb R}^{n}) } a_{-L}^{2\alpha }\sum _{j=-M}^{-L} {a_j^{2\alpha }\over a_{-L}^{2\alpha }} \\&\le C_{n, \alpha , v, \rho }\Vert \varphi \Vert _{L^\infty ({\mathbb R}^{n}) } {\rho ^{2\alpha }\over \rho ^{2\alpha }-1}a_{-L}^{2\alpha } \longrightarrow 0,\quad \hbox {as}\ { L,M \rightarrow +\infty }. \end{aligned}$$

As the set of test functions is dense in \(L^p(\mathbb {R}^n)\), by Theorem 1 we get the a.e. convergence for any function in \(L^p(\mathbb {R}^n)\). Analogously, since \(L^p(\mathbb {R}^n) \cap L^p(\mathbb {R}^n, \omega ) \) is dense in \(L^p(\mathbb {R}^n, \omega )\), we get the a.e. convergence for functions in \(L^p(\mathbb {R}^n, \omega ) \) with \(1\le p<\infty \). By using the dominated convergence theorem, we can prove the convergence in \(L^p(\mathbb {R}^n, \omega )\)-norm for \(1<p<\infty \), and also in measure. \(\square \)

3 Proof of Theorem 2

The dichotomy results announced in Theorem 1, parts (c) and (d), about \(L^\infty (\mathbb R^{n})\) and \(BMO(\mathbb R^{n})\) are motivated, in part, by the existence of a bounded function f such that \(T^*f(x)=\infty \) as the following theorem shows. In [5], we can find some related results for the variation operators.

Theorem 9

There exist bounded sequence \(\{v_j\}_{j\in \mathbb Z}\), \(\rho \)-lacunary sequence \(\{a_j\}_{j\in \mathbb Z}\) and \(f\in L^\infty (\mathbb R^n)\) such that \(T^* f(x) =\infty \) for all \(x\in \mathbb R^n\).

Proof

We will only consider the case \(n=1\). For the multi-dimensional case, it is similar just with minor modifications. Let f be the function defined by

$$\begin{aligned} f(x) = \sum _{k\in \mathbb {Z}} (-1)^{k} \chi _{(-a^{2k+1}, -a^{2k}]}(x), \quad x\in {\mathbb R}, \end{aligned}$$

where \(a>1\) is a real number that we shall fix it later. It is easy to see that

$$\begin{aligned} f(a^{2j}x) = (-1)^j f(x). \end{aligned}$$
(3.1)

By changing variable, we have

$$\begin{aligned} \mathcal {P}_{a_j}^\alpha f(x)&= \frac{1}{4^\alpha \Gamma (\alpha ) } \int _0^{+\infty } \int _{\mathbb R}\frac{a_j^{2\alpha } e^{-a_j^{2}/(4s)}}{s^{1+\alpha }}{e^{-|y|^2/4s} \over (4\pi s)^{1/2}}f(x-y){\textrm{d}}y {\textrm{d}}s \\&=\frac{1}{\sqrt{\pi }\Gamma (\alpha ) } \int _0^{+\infty } s^\alpha e^{-s}\int _{\mathbb R}s^{1/2} e^{-s|y|^2} f(x-a_j y){\textrm{d}}y \frac{{\textrm{d}}s}{s}. \end{aligned}$$

Let \(a_j= a^{2j}.\) Then,

$$\begin{aligned} \mathcal {P}_{a_j}^\alpha f(0)&=\frac{1}{\sqrt{\pi }\Gamma (\alpha ) } \int _0^{+\infty } s^\alpha e^{-s}\int _{\mathbb R}s^{1/2} e^{-s|y|^2} f(-a^{2j} y){\textrm{d}}y \frac{{\textrm{d}}s}{s} \\&= \frac{(-1)^j}{\sqrt{\pi }\Gamma (\alpha ) } \int _0^{+\infty } s^\alpha e^{-s}\int _0^{+\infty } s^{1/2} e^{-s|y|^2} f(- y){\textrm{d}}y \frac{{\textrm{d}}s}{s}. \end{aligned}$$

We observe that

$$\begin{aligned} \int _0^{+\infty } s^{1/2} e^{-s|y|^2} \left| f(- y)\right| {\textrm{d}}y\le \int _0^{+\infty } s^{1/2} e^{-s|y|^2} {\textrm{d}}y=\int _0^{+\infty } e^{-t^2} {\textrm{d}}t={\sqrt{\pi }\over 2}. \end{aligned}$$

Therefore,

$$\begin{aligned} \int _0^{+\infty } s^\alpha e^{-s}\int _0^{+\infty } s^{1/2} e^{-s|y|^2}\left| f(- y)\right| {\textrm{d}}y \frac{{\textrm{d}}s}{s}\le {\sqrt{\pi }\over 2}\int _0^{+\infty } s^\alpha e^{-s}\frac{{\textrm{d}}s}{s}={\sqrt{\pi }\over 2}\Gamma (\alpha )< \infty . \end{aligned}$$

Also, we have

$$\begin{aligned}\lim _{R\rightarrow {+\infty }} \int _0^{+\infty }s^\alpha e^{-s}\int _R^{+\infty } s^{1/2} e^{-s|y|^2} f(- y){\textrm{d}}y \frac{{\textrm{d}}s}{s}=0\end{aligned}$$

and

$$\begin{aligned}\lim _{\varepsilon \rightarrow 0^+} \int _0^{+\infty }s^\alpha e^{-s}\int _0^\varepsilon s^{1/2} e^{-s|y|^2} f(- y){\textrm{d}}y\frac{{\textrm{d}}s}{s}=0. \end{aligned}$$

On the other hand, there exists a constant \(C>0\) such that

$$\begin{aligned}{} & {} \displaystyle \lim _{a\rightarrow {+\infty }}\int _0^{+\infty }s^\alpha e^{-s} \int _1^a s^{1/2} e^{-s|y|^2} f(- y){\textrm{d}}y\frac{{\textrm{d}}s}{s}\\ {}{} & {} \quad = \displaystyle \lim _{a\rightarrow {+\infty }}\int _0^{+\infty }s^\alpha e^{-s} \int _1^a s^{1/2} e^{-s|y|^2} {\textrm{d}}y\frac{{\textrm{d}}s}{s}=C>0.\end{aligned}$$

Hence we can choose \(a>1\) big enough such that

$$\begin{aligned}&\int _0^{+\infty }s^\alpha e^{-s}\int _1^a s^{1/2} e^{-s|y|^2} f(- y){\textrm{d}}y\frac{{\textrm{d}}s}{s} = \int _0^{+\infty }s^\alpha e^{-s}\int _1^a s^{1/2} e^{-s|y|^2} {\textrm{d}}y\frac{{\textrm{d}}s}{s} \\&> \int _0^{+\infty }s^\alpha e^{-s}{\int _0^{1/a} s^{1/2} e^{-s|y|^2} {\textrm{d}}y}\frac{{\textrm{d}}s}{s} + \int _0^{+\infty }s^\alpha e^{-s}{\int _{a^2}^{+\infty } s^{1/2} e^{-s|y|^2} {\textrm{d}}y}\frac{{\textrm{d}}s}{s}\\&> \int _0^{+\infty }s^\alpha e^{-s} \left| \int _0^{1/a} s^{1/2} e^{-s|y|^2} f(- y){\textrm{d}}y\right| \frac{{\textrm{d}}s}{s}\\ {}&\quad +\int _0^{+\infty }s^\alpha e^{-s} \left| \int _{a^2}^{+\infty } s^{1/2} e^{-s|y|^2} f(- y){\textrm{d}}y\right| \frac{{\textrm{d}}s}{s}. \end{aligned}$$

In other words, with the \(a>1\) fixed above, there exists constant \(C_1>0\) such that

$$\begin{aligned} \int _0^{+\infty }s^\alpha e^{-s}\int _0^{+\infty } s^{1/2} e^{-s|y|^2} f(- y){\textrm{d}}y\frac{{\textrm{d}}s}{s}=C_1. \end{aligned}$$
(3.2)

Hence

$$\begin{aligned} \Big | \mathcal {P}_{a_{j+1}}^\alpha f(0)-\mathcal {P}_{a_j}^\alpha f(0)\Big |= {{2 C_1}\over {\sqrt{\pi }\Gamma (\alpha )}}>0. \end{aligned}$$

Therefore, we have

$$\begin{aligned}\sum _{j\in \mathbb Z} \Big |\mathcal {P}_{a_{j+1}}^\alpha f(0)- \mathcal {P}_{a_{j}}^\alpha f(0)\Big | = \infty .\end{aligned}$$

By using (3.1) and changing variable we get

$$\begin{aligned} \mathcal {P}_{a_j}^\alpha f(x)&= \frac{1}{\sqrt{\pi }\Gamma (\alpha ) } \int _0^{+\infty } s^\alpha e^{-s}\int _{\mathbb R}s^{1/2} e^{-s|y|^2} f(x-a^{2j} y){\textrm{d}}y \frac{{\textrm{d}}s}{s}\\&=\frac{(-1)^j}{\sqrt{\pi }\Gamma (\alpha ) } \int _0^{+\infty } s^\alpha e^{-s}\int _{\mathbb R}s^{1/2} e^{-s|y|^2} f\left( {x\over a^{2j}}- y\right) {\textrm{d}}y \frac{{\textrm{d}}s}{s}. \end{aligned}$$

Then

$$\begin{aligned}&{\mathcal {P}_{a_{j+1}}^\alpha f(t)-\mathcal {P}_{a_j}^\alpha f(t)}\nonumber \\&= \frac{ (-1)^{j+1}}{\sqrt{\pi }\Gamma (\alpha ) } \Big \{\int _0^{+\infty } s^\alpha e^{-s}\int _{\mathbb R}s^{1/2} e^{-s|y|^2} f\left( {x\over a^{2(j+1)}}- y\right) {\textrm{d}}y \frac{{\textrm{d}}s}{s}\nonumber \\&\quad +\int _0^{+\infty } s^\alpha e^{-s}\int _{\mathbb R}s^{1/2} e^{-s|y|^2} f\left( {x\over a^{2j}}- y\right) {\textrm{d}}y \frac{{\textrm{d}}s}{s}\Big \}. \end{aligned}$$
(3.3)

By the dominated convergence theorem, we know that

$$\begin{aligned}&\lim _{h\rightarrow 0}\int _0^{+\infty } s^\alpha e^{-s}\int _{\mathbb R}s^{1/2} e^{-s|y|^2} f\left( h- y\right) {\textrm{d}}y\frac{{\textrm{d}}s}{s}\\&=\int _0^{+\infty } s^\alpha e^{-s}\int _{\mathbb R}s^{1/2} e^{-s|y|^2} f\left( - y\right) {\textrm{d}}y \frac{{\textrm{d}}s}{s} \\&=\int _0^{+\infty } s^\alpha e^{-s}\int _0^{+\infty }s^{1/2} e^{-s|y|^2} f\left( - y\right) {\textrm{d}}y\frac{{\textrm{d}}s}{s} \\&= C_1>0, \end{aligned}$$

where \(C_1\) is the constant appeared in (3.2). So, there exists \(0<\eta _0<1,\) such that, for \(|h|<\eta _0,\)

$$\begin{aligned} \int _0^{+\infty } s^\alpha e^{-s}\int _{\mathbb R}s^{1/2} e^{-s|y|^2} f\left( h- y\right) {\textrm{d}}y\frac{{\textrm{d}}s}{s}\ge \frac{C_1}{2}. \end{aligned}$$

Then, for each \(x\in {\mathbb R}\), we can choose \(j\in \mathbb Z\) such that \(\displaystyle {|x|\over {a^j}}<\eta _0\) (there are infinite j satisfying this condition), and we have

$$\begin{aligned}{} & {} \int _0^{+\infty } s^\alpha e^{-s}\int _{\mathbb R}s^{1/2} e^{-s|y|^2} f\left( {x\over a^{2(j+1)}}- y\right) {\textrm{d}}y\frac{{\textrm{d}}s}{s} \\{} & {} \quad +\int _0^{+\infty } s^\alpha e^{-s}\int _{\mathbb R}s^{1/2} e^{-s|y|^2} f\left( {x\over a^{2j}}- y\right) {\textrm{d}}y \frac{{\textrm{d}}s}{s} \ge C_1>0. \end{aligned}$$

Choosing \(v_j=(-1)^{j+1},\ j\in \mathbb Z\), by (3.3) we have, for any \(x\in \mathbb R,\)

$$\begin{aligned} T^* f(x)&\ge \sum _{\left| x\over {a^j}\right|<\eta _0} (-1)^{j+1}\big ({\mathcal P}_{a_{j+1}}^\alpha f(x)-{\mathcal P}_{a_j}^\alpha f(x)\big )\\&=\frac{ 1}{\sqrt{\pi }\Gamma (\alpha ) }\sum _{\left| x\over {a^j}\right|<\eta _0} \Big \{\int _0^{+\infty } s^\alpha e^{-s}\int _{\mathbb R}s^{1/2} e^{-s|y|^2} f\left( {x\over a^{2(j+1)}}- y\right) {\textrm{d}}y \frac{{\textrm{d}}s}{s}\\&\quad +\int _0^{+\infty } s^\alpha e^{-s}\int _{\mathbb R}s^{1/2} e^{-s|y|^2} f\left( {x\over a^{2j}}- y\right) {\textrm{d}}y \frac{{\textrm{d}}s}{s}\Big \}\\&\ge \frac{ 1}{\sqrt{\pi }\Gamma (\alpha ) }\sum _{\left| x\over {a^j}\right| <\eta _0}C_1=\infty . \end{aligned}$$

We complete the proof of Theorem 9. \(\square \)

At the end of this section, we will give the proof of Theorem 2.

Proof of Theorem 2

First, we prove the theorem in the case \(1<p<\infty .\) Since \(2r<1,\) we know that \(B\backslash B_{2r}\ne \emptyset .\) Let \(f(x)=f_1(x)+f_2(x)\), where \(f_1(x)=f(x)\chi _{B_{2r}}(x)\) and \(f_2(x)=f(x)\chi _{B\backslash B_{2r}}(x)\). Then

$$\begin{aligned}\left| T^* f(x)\right| \le \left| T^* f_1(x)\right| +\left| T^* f_2(x)\right| .\end{aligned}$$

By Theorem 1,

$$\begin{aligned} \frac{1}{|B_r|} \int _{B_r} \left| T^* f_1 (x)\right| {\textrm{d}}x\le & {} \left( \frac{1}{|B_r|} \int _{B_r} \left| T^* f_1 (x)\right| ^2 {\textrm{d}}x\right) ^{1/2}\\\le & {} C \left( \frac{1}{|B_r|} \int _{\mathbb {R}}\left| f_1 (x)\right| ^2 {\textrm{d}}x\right) ^{1/2}\le C\left\| f\right\| _{L^\infty (\mathbb {R}^n)}. \end{aligned}$$

We also know that, for any \(j\in \mathbb Z,\)

$$\begin{aligned}{} & {} \int _0^{+\infty }\int _{{\mathbb R}^n}\left| \frac{ a_{j+1}^{2\alpha }e^{-a_{j+1}^2/(4 s)}-a_j^{2\alpha } e^{-a_j^2/(4 s)}}{s^{1+\alpha }} \right| {e^{-|x-y|^2/4s} \over (4\pi s)^{n/2}}~ {\textrm{d}}y{\textrm{d}}s\nonumber \\{} & {} \quad \le \int _0^{+\infty }\int _{{\mathbb R}^n}{\frac{ a_{j+1}^{2\alpha }e^{-a_{j+1}^2/(4 s)}+a_j^{2\alpha } e^{-a_j^2/(4 s)}}{s^{1+\alpha }} }{e^{-|x-y|^2/4s} \over (4\pi s)^{n/2}}~ {\textrm{d}}y{\textrm{d}}s =2\cdot 4^\alpha \Gamma (\alpha ).\nonumber \\ \end{aligned}$$
(3.4)

Then, by Hölder’s inequality, (3.4) and Fubini’s Theorem, for \(1< p < \infty \) and any \(N=(N_1, N_2)\), we have

$$\begin{aligned}&\left| \sum _{j=N_1}^{N_2}v_j\left( {\mathcal P}_{a_{j+1}}^\alpha f_2(x)-{\mathcal P}_{a_j}^\alpha f_2(x)\right) \right| \\&\le C\sum _{j=N_1}^{N_2} \left| v_j \int _0^{+\infty }\frac{ a_{j+1}^{2\alpha }e^{-a_{j+1}^2/(4 s)}-a_j^{2\alpha } e^{-a_j^2/(4 s))}}{s^{1+\alpha }}\int _{{\mathbb R}^n}{e^{-|x-y|^2/4s} \over (4\pi s)^{n/2}} f_2(y)~{\textrm{d}}y {\textrm{d}}s\right| \\&\le C\left\| v\right\| _{l^p(\mathbb Z)}\left( \sum _{j=N_1}^{N_2}\left( \int _0^{+\infty }\left| \frac{ a_{j+1}^{2\alpha }e^{-a_{j+1}^2/(4 s)}-a_j^{2\alpha } e^{-a_j^2/(4 s)}}{s^{1+\alpha }} \right| \int _{{\mathbb R}^n}{e^{-|x-y|^2/4s} \over (4\pi s)^{n/2}}\left| f_2(y)\right| ~{\textrm{d}}y {\textrm{d}}s\right) ^{p'}\right) ^{1/p'} \\&\le C\left\| v\right\| _{l^p(\mathbb Z)}\Big (\sum _{j=N_1}^{N_2} \Big \{\int _0^{+\infty }\int _{{\mathbb R}^n}\left| \frac{ a_{j+1}^{2\alpha }e^{-a_{j+1}^2/(4 s)}-a_j^{2\alpha } e^{-a_j^2/(4 s)}}{s^{1+\alpha }} \right| {e^{-|x-y|^2/4s} \over (4\pi s)^{n/2}}\left| f_2(y)\right| ^{p'}~{\textrm{d}}y {\textrm{d}}s\Big \} \\&\quad \quad \times \Big \{\int _0^{+\infty }\int _{{\mathbb R}^n}\left| \frac{ a_{j+1}^{2\alpha }e^{-a_{j+1}^2/(4 s)}-a_j^{2\alpha } e^{-a_j^2/(4 s)}}{s^{1+\alpha }} \right| {e^{-|x-y|^2/4s} \over (4\pi s)^{n/2}}~ {\textrm{d}}y{\textrm{d}}s \Big \}^{p'/p} \Big )^{1/p'} \\&\le C \left\| v\right\| _{l^p(\mathbb Z)}\left( \sum _{j=N_1}^{N_2} \int _0^{+\infty }\int _{{\mathbb R}^n}\left| \frac{ a_{j+1}^{2\alpha }e^{-a_{j+1}^2/(4 s)}-a_j^{2\alpha } e^{-a_j^2/(4 s)}}{s^{1+\alpha }} \right| {e^{-|x-y|^2/4s} \over (4\pi s)^{n/2}}\left| f_2(y)\right| ^{p'}~{\textrm{d}}y {\textrm{d}}s \right) ^{1/p'} \\&\le C \left\| v\right\| _{l^p(\mathbb Z)}\left( \int _0^{+\infty }\int _{{\mathbb R}^n}\sum _{j=-\infty }^{+\infty } \left| \frac{ a_{j+1}^{2\alpha }e^{-a_{j+1}^2/(4 s)}-a_j^{2\alpha } e^{-a_j^2/(4 s)}}{s^{1+\alpha }} \right| {e^{-|x-y|^2/4s} \over (4\pi s)^{n/2}}\left| f_2(y)\right| ^{p'}~{\textrm{d}}y {\textrm{d}}s \right) ^{1/p'}\\&\le C \left\| v\right\| _{l^p(\mathbb Z)}\left( \int _0^{+\infty } \int _{{\mathbb R}^n}{1\over s} {e^{-|x-y|^2/4s} \over (4\pi s)^{n/2}}\left| f_2(y)\right| ^{p'}~{\textrm{d}}y {\textrm{d}}s \right) ^{1/p'}. \end{aligned}$$

For \(x\in B\backslash B_{2r}\) and \(y\in B_r\), we have \(r\le |x-y|\le 2\). Then, by integration with polar coordinates we get

$$\begin{aligned} \frac{1}{|B_r|} \int _{B_r} \left| T^* f_2(x)\right| {\textrm{d}}x&\le C\frac{1}{|B_r|} \int _{B_r} \left( \int _0^{+\infty } \int _{{\mathbb R}^n}{1\over s} {e^{-|x-y|^2/4s} \over (4\pi s)^{n/2}}\left| f_2(y)\right| ^{p'}~{\textrm{d}}y {\textrm{d}}s \right) ^{1/p'}{\textrm{d}}t \\&\le C\frac{\left\| f\right\| _{L^\infty ({\mathbb R}^n)}}{|B_r|} \int _{B_r} \left( \int _0^{+\infty }\int _{S^{n-1}}\int _{r\le |t| \le 2} \frac{1}{s} {e^{-t^2/4s} \over (4\pi s)^{n/2}}~{\textrm{d}}td_{\omega _{n-1}} {\textrm{d}}s \right) ^{1/p'}{\textrm{d}}t \\&\sim \Big (\log \frac{2}{r}\Big )^{1/p'}\left\| f\right\| _{L^\infty ({\mathbb R}^n)}. \end{aligned}$$

Hence,

$$\begin{aligned}\frac{1}{|B_r|} \int _{B_r} \left| T^* f(x)\right| {\textrm{d}}x\le & {} C\left( 1+\Big (\log \frac{2}{r}\Big )^{1/p'}\right) \left\| f\right\| _{L^\infty ({\mathbb R}^n)}\\\le & {} C\Big (\log \frac{2}{r}\Big )^{1/p'}\left\| f\right\| _{L^\infty ({\mathbb R}^n)}.\end{aligned}$$

For the case \(p=1\) and \(p=\infty \), the proof is similar and easier. Then we get the proof of (a).

For (b),  we only consider the case \(n=1\). It is similar in the multi-dimensional case. When \(1< p<\infty ,\) for any \(0<\varepsilon <p-1\), let

$$\begin{aligned}\displaystyle f(x) = \sum _{k=-\infty }^{0} (-1)^{k} \chi _{(-a^{2k}, -a^{2k-1}]}(x) \,\, \hbox {and }\, \, a_j=a^{2j}, \end{aligned}$$

with \(a>1\) being fixed later. Then, the support of f is contained in \([-1, 0),\) and \(\{a_j\}_{j\in \mathbb Z}\) is a \(\rho \)-lacunary sequence with \(\rho =a^2>1.\) We observe that

$$\begin{aligned}{} & {} \left| \int _0^{+\infty } s^\alpha e^{-s}\int _0^{+\infty } s^{1/2} e^{-s|y|^2} f(-y){\textrm{d}}y\ \frac{{\textrm{d}}s}{s}\right| \le \int _0^{+\infty } s^\alpha e^{-s}\int _0^{+\infty } s^{1/2} e^{-s|y|^2} {\textrm{d}}y\frac{{\textrm{d}}s}{s}\\ {}{} & {} \quad ={\sqrt{\pi }\over 2}\Gamma (\alpha )< \infty . \end{aligned}$$

Hence

$$\begin{aligned}\lim _{R\rightarrow {+\infty }} \int _0^{+\infty } s^\alpha e^{-s}\int _R^{+\infty } s^{1/2} e^{-s|y|^2} f(-y){\textrm{d}}y =0\end{aligned}$$

and

$$\begin{aligned}\lim _{\varepsilon \rightarrow 0^+} \int _0^{+\infty } s^\alpha e^{-s}\int _0^\varepsilon s^{1/2} e^{-s|y|^2} f(-y){\textrm{d}}y\frac{{\textrm{d}}s}{s}=0. \end{aligned}$$

Also there exists a constant \(C>0\) such that

$$\begin{aligned}{} & {} \displaystyle \lim _{a\rightarrow {+\infty }} \int _0^{+\infty } s^\alpha e^{-s}\int _{a^{-1}}^1 s^{1/2} e^{-s|y|^2} f(-y){\textrm{d}}y\frac{{\textrm{d}}s}{s} \\ {}{} & {} \quad = \displaystyle \lim _{a\rightarrow {+\infty }}\int _0^{+\infty } s^\alpha e^{-s} \int _{a^{-1}}^1 s^{1/2} e^{-s|y|^2} {\textrm{d}}y\frac{{\textrm{d}}s}{s} =C.\end{aligned}$$

So, we can choose \(a>1\) big enough such that

$$\begin{aligned}&\int _0^{+\infty } s^\alpha e^{-s}\int _{a^{-1}}^1 s^{1/2} e^{-s|y|^2} f(-y){\textrm{d}}y\frac{{\textrm{d}}s}{s} = \int _0^{+\infty } s^\alpha e^{-s}\int _{a^{-1}}^1 s^{1/2} e^{-s|y|^2} {\textrm{d}}y\frac{{\textrm{d}}s}{s}\\&\ge 10 \left( \int _0^{+\infty } s^\alpha e^{-s}\int _0^{1/a^2} s^{1/2} e^{-s|y|^2} {\textrm{d}}y+\int _0^{+\infty } s^\alpha e^{-s}\int _{a-1}^{+\infty } s^{1/2} e^{-s|y|^2} {\textrm{d}}y\frac{{\textrm{d}}s}{s}\right) \nonumber \\&> 10 \left( \left| \int _0^{+\infty } s^\alpha e^{-s}\int _0^{1/a^2} s^{1/2} e^{-s|y|^2} f(-y){\textrm{d}}y\frac{{\textrm{d}}s}{s}\right| \right. \\ {}&\quad \left. +\left| \int _0^{+\infty } s^\alpha e^{-s}\int _{a-1}^{+\infty } s^{1/2} e^{-s|y|^2} f(-y){\textrm{d}}y\frac{{\textrm{d}}s}{s}\right| \right) . \end{aligned}$$

Therefore, there exists a constant \(C_1>0\) such that

$$\begin{aligned} \int _0^{+\infty } s^\alpha e^{-s}\int _0^{+\infty }s^{1/2} e^{-s|y|^2} f(-y){\textrm{d}}y\frac{{\textrm{d}}s}{s}=C_1>0 \end{aligned}$$
(3.5)

and

$$\begin{aligned} 0< & {} \int _0^{+\infty } s^\alpha e^{-s}\int _0^{1/ a^2} s^{1/2} e^{-s|y|^2} {\textrm{d}}y\frac{{\textrm{d}}s}{s}\nonumber \\ {}{} & {} +\int _0^{+\infty } s^\alpha e^{-s}\int _{a-1}^{+\infty } s^{1/2} e^{-s|y|^2} {\textrm{d}}y\frac{{\textrm{d}}s}{s}\le {C_1\over 9}. \end{aligned}$$
(3.6)

On the other hand, by the dominated convergence theorem, we have

$$\begin{aligned}{} & {} \lim _{h\rightarrow 0}\int _0^{+\infty } s^\alpha e^{-s}\int _{\mathbb R}s^{1/2} e^{-s|y|^2} f(h-y){\textrm{d}}y\frac{{\textrm{d}}s}{s}\\ {}{} & {} \quad =\int _0^{+\infty } s^\alpha e^{-s}\int _0^{+\infty } s^{1/2} e^{-s|y|^2} f(-y){\textrm{d}}y\frac{{\textrm{d}}s}{s} = C_1>0, \end{aligned}$$

where \(C_1\) is the constant appeared in (3.5). So, there exists \(0<\eta _0<1,\) such that, for \(|h|<\eta _0,\)

$$\begin{aligned}{} & {} \int _0^{+\infty } s^\alpha e^{-s}\int _{\mathbb R}s^{1/2} e^{-s|y|^2} f(h-y){\textrm{d}}y\frac{{\textrm{d}}s}{s}\nonumber \\ {}{} & {} \quad \ge {1\over 2}\int _0^{+\infty } s^\alpha e^{-s}\int _0^{+\infty }s^{1/2} e^{-s|y|^2} f(-y){\textrm{d}}y\frac{{\textrm{d}}s}{s} = \frac{C_1}{2}. \end{aligned}$$
(3.7)

It can be checked that

$$\begin{aligned}f(a^{2j}x) = (-1)^j f(x)+(-1)^j\sum _{k=1}^{-j}(-1)^k \chi _{(-a^{2k}, -a^{2k-1}]}(x)\end{aligned}$$

when \(j\le 0.\) We will always assume \(j\le 0\) in the following. By changing variable,

$$\begin{aligned}{} & {} \mathcal {P}_{a_j}^\alpha f(x)= \frac{1}{\sqrt{\pi }\Gamma (\alpha ) } \int _0^{+\infty } s^\alpha e^{-s}\int _{\mathbb R}s^{1/2} e^{-s|y|^2} f\left( x-a^{2j} y\right) {\textrm{d}}y \frac{{\textrm{d}}s}{s}\\{} & {} \quad = \frac{(-1)^j }{\sqrt{\pi }\Gamma (\alpha ) } \int _0^{+\infty } s^\alpha e^{-s}\int _{\mathbb R}s^{1/2} e^{-s|y|^2}\\ {}{} & {} \quad \left\{ f\left( {x\over a^{2j}}- y\right) +\sum _{k=1}^{-j}(-1)^k \chi _{(-a^{2k}, -a^{2k-1}]}\left( {x\over a^{2j}}-y\right) \right\} {\textrm{d}}y \frac{{\textrm{d}}s}{s}. \end{aligned}$$

Then

$$\begin{aligned}&{\mathcal {P}_{a_{j+1}}^\alpha f(x)-\mathcal {P}_{a_j}^\alpha f(x)}\nonumber \\&=\frac{ (-1)^{j+1}}{\sqrt{\pi }\Gamma (\alpha ) } \Big \{\, \int _0^{+\infty } s^\alpha e^{-s}\int _{\mathbb R}s^{1/2} e^{-s|y|^2} f\left( {x\over a^{2j+2}}- y\right) {\textrm{d}}y \frac{{\textrm{d}}s}{s}\nonumber \\&\quad +\int _0^{+\infty } s^\alpha e^{-s}\int _{\mathbb R}s^{1/2} e^{-s|y|^2} f\left( {x\over a^{2j}}- y\right) {\textrm{d}}y \frac{{\textrm{d}}s}{s} \nonumber \\&\quad +\int _0^{+\infty } s^\alpha e^{-s}\int _{\mathbb R}s^{1/2} e^{-s|y|^2}\sum _{k=1}^{-j-1}(-1)^k \chi _{(-a^{2k}, -a^{2k-1}]}\left( {x\over a^{2j+2}}-y\right) {\textrm{d}}y \frac{{\textrm{d}}s}{s}\nonumber \\&\quad +\int _0^{+\infty } s^\alpha e^{-s}\int _{\mathbb R}s^{1/2} e^{-s|y|^2}\sum _{k=1}^{-j}(-1)^k \chi _{(-a^{2k}, -a^{2k-1}]}\left( {x\over a^{2j}}-y\right) {\textrm{d}}y \frac{{\textrm{d}}s}{s}\, \Big \}. \end{aligned}$$
(3.8)

For given \(\eta _0\) as above, let \(2r<1\) such that \(r< \eta _0^2\) and \(r \sim a^{2J_0}\eta _0\) for a certain negative integer \(J_0\). If \(J_0\le j\le 0\), we have \(\displaystyle {r\over a^{2j}} <\eta _0\). And, for any \(-r\le x\le r\) we have

$$\begin{aligned}\displaystyle -1\cdot \chi _{[a-1,+\infty )}(y)\le \sum _{k=1}^{-j-1}(-1)^k\chi _{(-a^{2k}, -a^{2k-1}]}\left( {x\over a^{2j+2}}-y\right) \le \chi _{[a-1,+\infty )}(y)\end{aligned}$$

and

$$\begin{aligned}\displaystyle -1\cdot \chi _{[a-1,+\infty )}(y) \le \sum _{k=1}^{-j} (-1)^k\chi _{(-a^{2k}, -a^{2k-1}]}\left( {x\over a^{2j}}-y\right) \le \chi _{[a-1,+\infty )}(y). \end{aligned}$$

Hence, for the third and fourth integrals in (3.8), by (3.6) we have

$$\begin{aligned}&\int _0^{+\infty } s^\alpha e^{-s}\int _{\mathbb R}s^{1/2} e^{-s|y|^2}\sum _{k=1}^{-j-1}(-1)^k \chi _{(-a^{2k}, -a^{2k-1}]}\left( {x\over a^{2j+2}}-y\right) {\textrm{d}}y \frac{{\textrm{d}}s}{s}\nonumber \\&\quad \quad \quad \quad + \int _0^{+\infty } s^\alpha e^{-s}\int _{\mathbb R}s^{1/2} e^{-s|y|^2}\sum _{k=1}^{-j}(-1)^k \chi _{(-a^{2k}, -a^{2k-1}]}\left( {x\over a^{2j}}-y\right) {\textrm{d}}y \frac{{\textrm{d}}s}{s}\nonumber \\&\quad \quad \quad \quad \quad \quad \quad \quad \quad \quad \quad \ge (-2)\int _0^{+\infty } s^\alpha e^{-s}\int _{a-1}^{+\infty } s^{1/2} e^{-s|y|^2}{\textrm{d}}y \frac{{\textrm{d}}s}{s}\ge -{2C_1\over 9}. \end{aligned}$$
(3.9)

So, for any \(x\in [-r, r]\) and \(J_0\le j\le 0\), combining (3.8), (3.7) and (3.9), we have

$$\begin{aligned} \left| \mathcal {P}_{a_{j+1}}^\alpha f(x)-\mathcal {P}_{a_j}^\alpha f(x)\right| \ge C_\alpha \cdot \left( C_1-{2C_1\over 9}\right) =C\cdot C_1>0. \end{aligned}$$

We choose the sequence \(\{v_j\}_{j\in \mathbb Z} \in \ell ^p(\mathbb Z)\) given by \(\displaystyle v_j=(-1)^{j+1}(-j)^{-{1\over p-\varepsilon }}\), then for \(N=(J_0, 0),\) we have

$$\begin{aligned}{} & {} \frac{1}{2r}\int _{[-r,r]} \left| T^* f(x)\right| {\textrm{d}}x\\ {}{} & {} \quad \ge \frac{1}{2r}\int _{[-r,r]} \left| T_N^\alpha f(x)\right| {\textrm{d}}x \ge \frac{1}{\sqrt{\pi }\Gamma (\alpha )} \frac{1}{2r}\int _{[-r,r]} \sum _{j =J_0}^{0} \left( C\cdot C_1\cdot (-j)^{-{1\over p-\varepsilon }}\right) {\textrm{d}}x\\{} & {} \quad \ge C_{p,\varepsilon ,\alpha }\cdot C_1\cdot (-J_0)^{1\over {(p-\varepsilon )'}} \sim \left( \log \frac{2}{r}\right) ^{1\over {(p-\varepsilon )'}}. \end{aligned}$$

For (c), let \(v_j=(-1)^{j+1}\), \(a_j=a^{2j}\) with \(a>1\) and \(0<\eta _0<1\) fixed in the proof of (b). Consider the same function f as in (b). Then, \(\left\| v\right\| _{l^\infty (\mathbb Z)}=1\) and \(\left\| f\right\| _{L^\infty (\mathbb R)}=1.\) By the same argument as in (b), with \(N=(J_0, 0)\) and \(0<\alpha <1\), we have

$$\begin{aligned}{} & {} \frac{1}{2r}\int _{[-r,r]} \left| T^* f(x)\right| {\textrm{d}}x\ge \frac{1}{2r}\int _{[-r,r]} \left| T_N^\alpha f(x)\right| {\textrm{d}}x\\{} & {} \quad \ge \frac{1}{\sqrt{\pi }\Gamma (\alpha )} \frac{1}{2r}\int _{[-r,r]} \sum _{j = J_0}^{0} C_1 {\textrm{d}}t \ge \frac{ C_1}{\sqrt{\pi }\Gamma (\alpha )}\cdot (-J_0) \sim \log \frac{2}{r}. \end{aligned}$$

\(\square \)

4 Boundedness of the differential transforms related to Schrödinger operator \(-\Delta +V\)

In this section, we would consider the differential transforms related with the generalized Poisson operators generated by the Schrödinger operator \({\mathcal L}=-\Delta +V\) in \({\mathbb R}^n\) with \(n\ge 3\), where the nonnegative potential V belongs to the reverse Hölder class \(RH_q\) with \(q\ge n/2\), that is, there exists \(C>0\), such that

$$\begin{aligned} \left( \frac{1}{|B|}\int _B V(x)^q{\textrm{d}}x\right) ^{\frac{1}{q}}\le \frac{C}{|B|}\int _B V(x){\textrm{d}}x, \end{aligned}$$

for every ball B in \({\mathbb R}^n\). Associated with this potential, Z. Shen defines the critical radii function in [16] as

$$\begin{aligned} \rho (x):=\sup \Big \{r>0:\frac{1}{r^{n-2}}\int _{B(x,r)}V(y)~{\textrm{d}}y\le 1\Big \},\quad x\in {\mathbb R}^n. \end{aligned}$$
(4.1)

We will abuse \(\rho \) in this article, and it should be easy to distinct the \(\rho \)-lacunary with \(\rho (x)\) for the reader. For more information related with Schödinger operators, see [8, 16].

Lemma 2

(See [16, Lemma 1.4]) There exist \(c>0\) and \(k_0\ge 1\) such that for all \(x,y\in {\mathbb R}^n\)

$$\begin{aligned} c^{-1}\rho (x)\left( 1+\frac{\left| x-y\right| }{\rho (x)}\right) ^{-k_0}\le \rho (y)\le c\rho (x) \left( 1+\frac{\left| x-y\right| }{\rho (x)}\right) ^{\frac{k_0}{k_0+1}}. \end{aligned}$$

In particular, there exists a positive constant \(C_1<1\) such that

$$\begin{aligned}\hbox {if}\quad \left| x-y\right| \le \rho (x)\quad \hbox {then}\quad C_1\rho (x)<\rho (y)<C_1^{-1}\rho (x).\end{aligned}$$

Let \(\left\{ {\mathcal T}_t\right\} _{t>0}\) be the heat–diffusion semigroup associated with \({\mathcal L}\):

$$\begin{aligned} {\mathcal T}_tf(x)\equiv e^{-t{\mathcal L}}f(x)=\int _{{\mathbb R}^n}e^{-t{\mathcal L}}(x,y)f(y)~{\textrm{d}}y,\qquad f\in L^2({\mathbb R}^n),~x\in {\mathbb R}^n,~t>0. \end{aligned}$$

Lemma 3

(See [9, 11]) For every \(M>0\) there exists a constant \(C_M\) such that

$$\begin{aligned} 0\le e^{-t{\mathcal L}}(x,y)\le C_M t^{-n/2}e^{-\frac{\left| x-y\right| ^2}{5t}}\left( 1+\frac{\sqrt{t}}{\rho (x)} +\frac{\sqrt{t}}{\rho (y)}\right) ^{-M},\quad x,y\in {\mathbb R}^n,~t>0. \end{aligned}$$

Lemma 4

(See [9, Proposition 2.16]) There exists a nonnegative Schwartz class function \(\omega \) on \({\mathbb R}^n\) such that

$$\begin{aligned} \left| e^{-t{\mathcal L}}(x,y)-W_t(x-y)\right| \le \left( \frac{\sqrt{t}}{\rho (x)}\right) ^{\delta _0}\omega _t(x-y),\quad x,y\in {\mathbb R}^n,~t>0, \end{aligned}$$

where \(W_t\) is the Gauss–Weierstrass kernel, \(\omega _t(x-y):=t^{-n/2}\omega \left( (x-y)/\sqrt{t}\right) \) and

$$\begin{aligned} \delta _0:=2-\frac{n}{q}>0. \end{aligned}$$
(4.2)

Lemma 5

(See [10, Proposition 4.11]) For every \(0<\delta <\delta _0,\) there exists a constant \(c>0\) such that for every \(M>0\) there exists a constant \(C>0\) such that for \(\left| x-y\right| <\sqrt{t}\) we have

$$\begin{aligned}\left| e^{-t{\mathcal L}}(x,z)-e^{-t{\mathcal L}}(y, z)\right| \le C\left( \frac{\left| x-y\right| }{\sqrt{t}}\right) ^\delta t^{-n/2}~e^{-c\left| x-z\right| ^2/t}\left( 1+\frac{\sqrt{t}}{\rho (x)}+\frac{\sqrt{t}}{\rho (z)}\right) ^{-M}.\end{aligned}$$

Lemma 6

(See [9, Proposition 2.17]) For every \(0<\delta <\min \{1,\delta _0\}\),

$$\begin{aligned}\left| \left( e^{-t{\mathcal L}}(x,z)-e^{t\Delta }(x-z)\right) -\left( e^{-t{\mathcal L}}(y,z)-e^{t\Delta }(y-z)\right) \right| \le C\left( \frac{\left| x-y\right| }{\rho (z)}\right) ^\delta \omega _t(x-z),\end{aligned}$$

for all \(x,z\in {\mathbb R}^n\) and \(t>0\), with \(\left| x-y\right| <C\rho (x)\) and \(\left| x-y\right| <\tfrac{1}{4}\left| x-z\right| \).

In fact, going through the proof of [9] we see that \(\omega (x)=e^{-\left| x\right| ^2}\).

Then by (1.1), we can define the generalized Poisson operators associated with the Schrödinger operator \({\mathcal L}\) as follows:

$$\begin{aligned} {\tilde{{\mathcal P}}}_t^\alpha f(x) =\frac{t^{2\alpha }}{4^\alpha \Gamma (\alpha )} \int _0^{+\infty } \int _{{\mathbb R}^n} {e^{-\frac{t^2}{4 s}}}e^{-s{\mathcal L}}(x,y) f (y){\textrm{d}}y \frac{{\textrm{d}}s}{s^{1+\alpha }}. \end{aligned}$$

Let \(\{a_j\}_{j\in {\mathbb Z}}\) be an increasing sequence of positive real numbers, and \(\{v_j\}_{j\in {\mathbb Z}}\) be a bounded sequence of real or complex numbers. We shall consider the differential transform series

$$\begin{aligned} \sum _{j\in {\mathbb Z}} v_j({\tilde{{\mathcal P}}}_{a_{j+1}}^\alpha f(x)-{\tilde{{\mathcal P}}}_{a_j}^\alpha f(x)). \end{aligned}$$

For each \(N\in {\mathbb Z}^2,~N=(N_1,N_2)\) with \(N_1<N_2,\) we define the sum

$$\begin{aligned} {{\tilde{T}}}_N^\alpha f(x)=\sum _{j=N_1}^{N_2} v_j({\tilde{{\mathcal P}}}_{a_{j+1}}^\alpha f(x)-{\tilde{{\mathcal P}}}_{a_j}^\alpha f(x)),\quad x\in \mathbb R^n. \end{aligned}$$
(4.3)

Then, we have the following formula:

$$\begin{aligned}&{{\tilde{T}}}_N^{\alpha } f(x) =\sum _{j=N_1}^{N_2} v_j({\tilde{{\mathcal P}}}_{a_{j+1}}^\alpha f(x)-{\tilde{{\mathcal P}}}_{a_j}^\alpha f(x)) \\&= \frac{1}{4^\alpha \Gamma (\alpha )} \sum _{j=N_1}^{N_2}v_j \int _{{\mathbb R}^n}\int _0^\infty \frac{ a_{j+1}^{2\alpha }e^{-a_{j+1}^2/(4 s)}-a_j^{2\alpha } e^{-a_j^2/(4 s)} }{s^{1+\alpha }} e^{-s{\mathcal L}}(x,y) f(y)~{\textrm{d}}s {\textrm{d}}y \\&= \frac{1}{4^\alpha \Gamma (\alpha )} \int _{{\mathbb R}^{n}} \int _0^{+\infty }\sum _{j=N_1}^{N_2}v_j \frac{ a_{j+1}^{2\alpha }e^{-a_{j+1}^2/(4 s)}-a_j^{2\alpha } e^{-a_j^2/(4 s)} }{s^{1+\alpha }} e^{-s{\mathcal L}}(x,y)~{\textrm{d}}s f(y) {\textrm{d}}y. \end{aligned}$$

We denote the kernel of \({{\tilde{T}}}_N^\alpha \) by

$$\begin{aligned}{{\tilde{K}}}_N^\alpha (x, y)=\frac{1}{4^\alpha \Gamma (\alpha )}\int _0^{+\infty } \sum _{j=N_1}^{N_2}v_j \frac{ a_{j+1}^{2\alpha }e^{-a_{j+1}^2/(4 s)}-a_j^{2\alpha } e^{-a_j^2/(4 s)}}{s^{1+\alpha }} e^{-s{\mathcal L}}(x,y){\textrm{d}}s.\end{aligned}$$

By Lemma 3, we can prove the following theorem as in the proof of Theorem 5, which indicate that the kernel \({{\tilde{K}}}_N^\alpha \) is an \(\ell ^\infty (\mathbb Z^2)\)-valued Calderón–Zygmund kernel.

Theorem 10

For any \(x, y\in {\mathbb R}^n,\) \(x\ne y\), and \(M>0,\) there exists constants C depending on \(n, M, \alpha \) and \(\left\| v\right\| _{l^\infty (\mathbb Z)}\) such that

  1. (i)

    \(\displaystyle \left| {{\tilde{K}}}_N^\alpha (x, y)\right| \le \frac{C}{|x-y|^{n}}\left( 1+\frac{|x-y|}{\rho (x)}+\frac{|x-y|}{\rho (y)}\right) ^{-M}\),

  2. (ii)

    \(\displaystyle | {{\tilde{K}}}_N^\alpha (x,y)- {{\tilde{K}}}_N^\alpha (x,z)|+|{{\tilde{K}}}_N^\alpha (y,x)-{{\tilde{K}}}_N^\alpha (z,x)| \le C\frac{\left| y-z\right| ^\delta }{|x-y|^{n+\delta }}\), whenever \(\left| x-y\right| >{2}\left| y-z\right| \), for any \(\displaystyle 0<\delta <2-{n\over q}.\)

Proof

(i) For any \(M>0,\) we have

$$\begin{aligned}{} & {} |{{\tilde{K}}}_N^\alpha (x, y)| \le C\left\| v\right\| _{l^\infty (\mathbb Z)}\int _0^{+\infty }\sum _{j=-\infty }^{\infty } \left| \frac{a_{j+1}^{2\alpha } e^{-a_{j+1}^2/(4s)}-a_j^{2\alpha } e^{-a_j^2/(4s)}}{s^{1+\alpha }} e^{-s{\mathcal L}}(x, y)\right| {\textrm{d}}s\\{} & {} \quad = C\int _0^{+\infty }\sum _{j=-\infty }^{\infty } \left| a_{j+1}^{2\alpha } e^{-a_{j+1}^2/(4s)}-a_j^{2\alpha } e^{-a_j^2/(4s)}\right| \frac{e^{-s{\mathcal L}}(x, y)}{s^{1+\alpha }}{\textrm{d}}s. \end{aligned}$$

Then, by (2.1) and Lemma 3 we have

$$\begin{aligned} |{{\tilde{K}}}_N^\alpha (x, y)|\le & {} C \int _0^{+\infty }\frac{e^{-s{\mathcal L}}(x, y)}{s}{\textrm{d}}s=C \left( \int _0^{|x-y|^2}+\int _{|x-y|^2}^{+\infty }\right) \frac{e^{-|x-y|^2/(cs)}}{s^{n/2+1}}{\textrm{d}}s\\\le & {} C \left( \int _0^{|x-y|^2}\frac{1}{|x-y|^{n+2}}{\textrm{d}}s+\int _{|x-y|^2}^{+\infty }\frac{1}{s^{n/2+1}}{\textrm{d}}s\right) \le \frac{C}{|x-y|^{n}}, \end{aligned}$$

and

$$\begin{aligned} |{{\tilde{K}}}_N^\alpha (x, y)|&\le C \int _0^{+\infty }\frac{e^{-s{\mathcal L}}(x, y)}{s}{\textrm{d}}s\\ {}&=C \left( \int _0^{|x-y|^2}+\int _{|x-y|^2}^{+\infty }\right) s^{-n/2-1}{e^{-|x-y|^2/(cs)}}\left( {\sqrt{s}\over \rho (x)}\right) ^{-M}{\textrm{d}}s\\&\le C \left( \int _0^{|x-y|^2}\frac{\rho ^M(x)}{|x-y|^{n+M+2}}{\textrm{d}}s+\int _{|x-y|^2}^{+\infty }\frac{\rho ^M(x)}{s^{n/2+M/2+1}}{\textrm{d}}s\right) \\&\le \frac{C}{|x-y|^{n}}\left( {|x-y|\over \rho (x)}\right) ^{-M}. \end{aligned}$$

Then, together with the symmetry of \(e^{-s{\mathcal L}}(x, y)\), we have

$$\begin{aligned}{{{\tilde{K}}}_N^\alpha (x, y)}\le \frac{C}{|x-y|^{n}}\left( 1+\frac{|x-y|}{\rho (x)}+\frac{|x-y|}{\rho (y)}\right) ^{-M}.\end{aligned}$$

(ii) It can be proved with the same method as in i) with Proposition 3.10 in [4]. \(\square \)

Theorem 11

For the operator \({{\tilde{T}}}_N^\alpha \) defined in (4.3), we have the following statements:

  1. (a)

    For any \(1<p<\infty \) and \(\omega \in A_p\), there exists a constant \(C>0\) depending on \(n, p, \alpha \) and \(\left\| v\right\| _{l^\infty (\mathbb Z)}\) such that

    $$\begin{aligned}\left\| {{\tilde{T}}}_N^\alpha f\right\| _{L^p(\mathbb R^{n}, \omega )}\le C\left\| f\right\| _{L^p(\mathbb R^{n}, \omega )},\end{aligned}$$

    for all functions \(f\in L^p({\mathbb R}^{n}, \omega ).\)

  2. (b)

    For any \(\omega \in A_1\), there exists a constant \(C>0\) depending on \(n, \alpha \) and \(\left\| v\right\| _{l^\infty (\mathbb Z)}\) such that

    $$\begin{aligned}\omega \left( {\{x\in {\mathbb R}^{n}:\left| {{\tilde{T}}}_N^\alpha f(x,t)\right|>\lambda \}}\right) \le C\frac{1}{\lambda }\left\| f\right\| _{L^1(\mathbb R^{n}, \omega )},\quad \lambda >0,\end{aligned}$$

    for all functions \(f\in L^1({\mathbb R}^{n}, \omega ).\)

The constants C appeared above all are independent with N.

Proof

For any \(f\in L^p({\mathbb R}^n, \omega )(1\le p<+\infty ),\) we have

$$\begin{aligned} \left| {{\tilde{T}}}_N^\alpha f(x)\right|&= \left| \int _{{\mathbb R}^n}{{\tilde{K}}}_N^\alpha (x, y)f(y){\textrm{d}}y\right| \\ {}&\le \left| \int _{\left| x-y\right| \le \rho (x)}{{\tilde{K}}}_N^\alpha (x, y)f(y){\textrm{d}}y\right| +\left| \int _{\left| x-y\right|> \rho (x)}{{\tilde{K}}}_N^\alpha (x, y)f(y){\textrm{d}}y\right| \\&\le \left| \int _{\left| x-y\right| \le \rho (x)}\left( {{\tilde{K}}}_N^\alpha (x, y)-K_N^\alpha (x-y)\right) f(y){\textrm{d}}y\right| \\ {}&+\left| \int _{\left| x-y\right| \le \rho (x)} K_N^\alpha (x, y)f(y){\textrm{d}}y\right| \\&\quad +\left| \int _{\left| x-y\right| > \rho (x)}{{\tilde{K}}}_N^\alpha (x, y)f(y){\textrm{d}}y\right| \\&=:I_1+I_2+I_3. \end{aligned}$$

For \(I_1,\) by (2.1) and Lemma  4, we get

$$\begin{aligned}&I_1=C\left| \int _{\left| x-y\right| \le \rho (x)}\int _0^{+\infty } \sum _{j=N_1}^{N_2}v_j \frac{ a_{j+1}^{2\alpha }e^{-a_{j+1}^2/(4 s)}-a_j^{2\alpha } e^{-a_j^2/(4 s)}}{s^{1+\alpha }} \left( e^{-s{\mathcal L}}(x,y)-W_s(x-y)\right) {\textrm{d}}sf(y){\textrm{d}}y\right| \\&\le C\left\| v\right\| _{l^\infty (\mathbb Z)}\int _{\left| x-y\right| \le \rho (x)}\int _0^{+\infty } \sum _{j=-\infty }^{+\infty }\left| \frac{ a_{j+1}^{2\alpha }e^{-a_{j+1}^2/(4 s)}-a_j^{2\alpha } e^{-a_j^2/(4 s)}}{s^{1+\alpha }}\right| \left( \frac{\sqrt{s}}{\rho (x)}\right) ^{\delta _0} \frac{ e^{-c\frac{\left| x-y\right| ^2}{4s}}}{s^{n/2}}{\textrm{d}}s|f(y)|{\textrm{d}}y\\&\le C\int _{\left| x-y\right| \le \rho (x)}\int _0^{+\infty } \frac{1}{s} \left( \frac{\sqrt{s}}{\rho (x)}\right) ^{\delta _0} \frac{ e^{-c\frac{\left| x-y\right| ^2}{4s}}}{s^{n/2}}{\textrm{d}}s|f(y)|{\textrm{d}}y\\&=C\int _{\left| x-y\right| \le \rho (x)}\int _0^{\rho ^2(x)}\frac{1}{s} \left( \frac{\sqrt{s}}{\rho (x)}\right) ^{\delta _0} \frac{ e^{-c\frac{\left| x-y\right| ^2}{4s}}}{s^{n/2}}{\textrm{d}}s|f(y)|{\textrm{d}}y\\&\quad +C\int _{\left| x-y\right| \le \rho (x)}\int _{\rho ^2(x)}^{+\infty } \frac{1}{s} \left( \frac{\sqrt{s}}{\rho (x)}\right) ^{\delta _0} \frac{ e^{-c\frac{\left| x-y\right| ^2}{4s}}}{s^{n/2}}{\textrm{d}}s|f(y)|{\textrm{d}}y\\&=:I_{11}+I_{12}. \end{aligned}$$

For the term \(I_{11}\), we have

$$\begin{aligned} I_{11}&=C\int _{\left| x-y\right| \le \rho (x)}\int _0^{\rho ^2(x)}{s^{\frac{\delta _0-n}{2}-1}}{\rho (x)^{-\delta _0}} { e^{-c\frac{\left| x-y\right| ^2}{4s}}}{\textrm{d}}s|f(y)|{\textrm{d}}y\\&\le C{\rho (x)^{-\delta _0}}\int _0^{\rho ^2(x)}{s^{\frac{\delta _0}{2}-1}}\cdot \frac{1}{s^{{n}/{2}}}\int _{{\mathbb R}^n} { e^{-c\frac{\left| x-y\right| ^2}{4s}}}|f(y)|{\textrm{d}}y {\textrm{d}}s\\&\le C \sup _{s>0} \frac{1}{s^{{n}/{2}}}\int _{{\mathbb R}^n} { e^{-c\frac{\left| x-y\right| ^2}{4s}}}|f(y)|{\textrm{d}}y\\&\le C {\mathcal M}(f)(x), \quad x\in {\mathbb R}^n, \end{aligned}$$

where \({\mathcal M}\) denotes the classical Hardy–Littlewood maximal function.

For the term \(I_{12}\), since \(\displaystyle \delta _0=2-\frac{n}{q}\)(see (4.2)) and \(n\ge 3,\) we have

$$\begin{aligned} I_{12}&\le C\int _{\left| x-y\right| \le \rho (x)}\int _{\rho ^2(x)}^{+\infty }{s^{\frac{\delta _0-n}{2}-1}}{\rho (x)^{-\delta _0}} {\textrm{d}}s|f(y)|{\textrm{d}}y\\&\le C{\rho (x)^{-n}}\int _{\left| x-y\right| \le \rho (x)}|f(y)|{\textrm{d}}y \\&\le C {\mathcal M}(f)(x), \quad x\in {\mathbb R}^n. \end{aligned}$$

Hence, we have

$$\begin{aligned}I_1\le C{\mathcal M}(f)(x), \quad x\in {\mathbb R}^n.\end{aligned}$$

For \(I_2,\) we can write

$$\begin{aligned} I_2&=\left| \int _{\left| x-y\right| \le \rho (x)} K_N^\alpha (x, y)f(y){\textrm{d}}y\right| \le \left| \int _{{\mathbb R}^n} K_N^\alpha (x, y)f(y){\textrm{d}}y\right| \\ {}&\quad +\sup _{\varepsilon>0}\left| \int _{\left| x-y\right| >\varepsilon } K_N^\alpha (x, y)f(y){\textrm{d}}y\right| . \end{aligned}$$

Now, let us consider the operator defined by

$$\begin{aligned} T:L^2({\mathbb R}^n)&\longrightarrow L^2({\mathbb R}^n)\\ f&\mapsto Tf(x)=\int _{{\mathbb R}^n}K_N^\alpha (x,y) f(y){\textrm{d}}y. \end{aligned}$$

From Theorem 4, we know that T is bounded on \(L^2({\mathbb R}^n)\). And T is a Calderón–Zygmund operator associated with the kernel \(K_N^\alpha (x,y)\)(see Theorem 5). Then, by proving a Cotlar’s inequality as in [18, p. 34, Proposition 2] and the argument in [18, p. 36, Corollary 2], we can prove that the maximal operator \(T^{\alpha ,*}_N\) defined by

$$\begin{aligned}T^{\alpha ,*}_Nf(x)=\sup _{\varepsilon>0}\left| \int _{\left| x-y\right| >\varepsilon }K_N^\alpha (x,y)f(y){\textrm{d}}y\right| \end{aligned}$$

is bounded on \(L^p({\mathbb R}^n, \omega )\), for every \(1<p<\infty \), and from \(L^1({\mathbb R}^n, \omega )\) in to \(L^{1,\infty }({\mathbb R}^n, \omega ).\) Combining this fact with Theorem 3, we conclude that

$$\begin{aligned}\left\| I_2\right\| _{L^p({\mathbb R}^n, \omega )}\le C\left\| f\right\| _{L^p({\mathbb R}^n, \omega )},\quad 1<p<\infty \end{aligned}$$

and

$$\begin{aligned}\left\| I_2\right\| _{L^{1,\infty }({\mathbb R}^n, \omega )}\le C\left\| f\right\| _{L^1({\mathbb R}^n, \omega )}.\end{aligned}$$

For the last term \(I_3,\) by (2.1) and Lemma 3 with \(M>1\),

$$\begin{aligned} I_3&=\left| \int _{\left| x-y\right|> \rho (x)}{{\tilde{K}}}_N^\alpha (x, y)f(y){\textrm{d}}y\right| \\&=C\left| \int _{\left| x-y\right|> \rho (x)}\int _0^{+\infty } \sum _{j=N_1}^{N_2}v_j \frac{ a_{j+1}^{2\alpha }e^{-a_{j+1}^2/(4 s)}-a_j^{2\alpha } e^{-a_j^2/(4 s)}}{s^{1+\alpha }} e^{-s{\mathcal L}}(x,y){\textrm{d}}sf(y){\textrm{d}}y\right| \\&\le C\left\| v\right\| _{l^\infty (\mathbb Z)}\int _{\left| x-y\right|> \rho (x)}\int _0^{+\infty } \sum _{j=-\infty }^{+\infty }\left| \frac{ a_{j+1}^{2\alpha }e^{-a_{j+1}^2/(4 s)}-a_j^{2\alpha } e^{-a_j^2/(4 s)}}{s^{1+\alpha }}\right| e^{-s{\mathcal L}}(x,y){\textrm{d}}s|f(y)|{\textrm{d}}y\\&\le C \int _{\left| x-y\right|> \rho (x)}\int _0^{+\infty }\frac{1}{ s^{\frac{n}{2}+1}}e^{-\frac{\left| x-y\right| ^2}{5s}} \left( 1+\frac{\sqrt{s}}{\rho (x)} +\frac{\sqrt{s}}{\rho (y)}\right) ^{-M}{\textrm{d}}s|f(y)|{\textrm{d}}y\\&= C \int _{\left| x-y\right|> \rho (x)}\int _0^{\rho ^2(x)}\frac{1}{ s^{\frac{n}{2}+1}}e^{-\frac{\left| x-y\right| ^2}{5s}} \left( 1+\frac{\sqrt{s}}{\rho (x)} +\frac{\sqrt{s}}{\rho (y)}\right) ^{-M}{\textrm{d}}s|f(y)|{\textrm{d}}y\\&\quad +C\int _{\left| x-y\right| > \rho (x)}\int _{\rho ^2(x)}^{+\infty }\frac{1}{ s^{\frac{n}{2}+1}} e^{-\frac{\left| x-y\right| ^2}{5s}}\left( 1+\frac{\sqrt{s}}{\rho (x)} +\frac{\sqrt{s}}{\rho (y)}\right) ^{-M}{\textrm{d}}s|f(y)|{\textrm{d}}y\\&=:I_{31}+I_{32}. \end{aligned}$$

For the term \(I_{31}\),

$$\begin{aligned} I_{31}&\le C\int _{\left| x-y\right|> \rho (x)}\int _0^{\rho ^2(x)}\frac{1}{ s^{\frac{n}{2}+1}}e^{-\frac{\left| x-y\right| ^2}{5s}} {\textrm{d}}s|f(y)|{\textrm{d}}y\\&\le C\int _0^{\rho ^2(x)}\frac{1}{s} e^{-c_1\frac{\rho ^2(x)}{s}}\left( \frac{1}{ s^{n /2}}\int _{{\mathbb R}^n}e^{-c_2\frac{\left| x-y\right| ^2}{s}}|f(y)|{\textrm{d}}y\right) {\textrm{d}}s\\&\le C\sup _{s>0}\frac{1}{ s^{n /2}}\int _{{\mathbb R}^n}e^{-c_2\frac{\left| x-y\right| ^2}{s}}|f(y)|{\textrm{d}}y\\&\le C{\mathcal M}(f)(x),\quad x\in {\mathbb R}^n. \end{aligned}$$

For the other term \(I_{32}\), by changing variable we have

$$\begin{aligned} I_{32}&\le C\int _{\left| x-y\right|> \rho (x)}\int _{\rho ^2(x)}^{+\infty }\frac{1}{ s^{\frac{n}{2}+1}} e^{-\frac{\left| x-y\right| ^2}{5s}}\left( 1+\frac{\sqrt{s}}{\rho (x)} \right) ^{-M}{\textrm{d}}s|f(y)|{\textrm{d}}y\\&=C\int _{\left| x-y\right|> \rho (x)}|f(y)|\int _{1}^{+\infty }\frac{1}{ (u\rho (x))^n} (1+u)^{-M}e^{-c\frac{\left| x-y\right| ^2}{u^2\rho ^2(x)}}\frac{{\textrm{d}}u}{u}{\textrm{d}}y\\&\le C\frac{1}{\rho ^n(x)}\int _{\left| x-y\right|> \rho (x)}|f(y)|\int _{1}^{+\infty }(1+u)^{-M}\left( \frac{u\rho (x)}{\left| x-y\right| }\right) ^{n+1}\frac{{\textrm{d}}u}{u^{n+1}}{\textrm{d}}y\\&\le C\frac{1}{\rho ^n(x)}\int _{\left| x-y\right| > \rho (x)}|f(y)|\left( \frac{\rho (x)}{\left| x-y\right| }\right) ^{n+1}{\textrm{d}}y\\&=C\frac{1}{\rho ^n(x)}\sum _{k=0}^{+\infty }\int _{2^k\rho (x)<\left| x-y\right| \le 2^{k+1} \rho (x)}|f(y)|\left( \frac{\rho (x)}{\left| x-y\right| }\right) ^{n+1}{\textrm{d}}y\\&\le C\sum _{k=0}^{+\infty }\frac{1}{2^k}\frac{1}{(2^k\rho (x))^n}\int _{\left| x-y\right| \le 2^{k+1} \rho (x)}|f(y)|{\textrm{d}}y\\&\le C{\mathcal M}(f)(x), \quad x\in {\mathbb R}^n. \end{aligned}$$

Hence, we get

$$\begin{aligned}I_3\le C{\mathcal M}(f)(x), \quad x\in {\mathbb R}^n.\end{aligned}$$

Then, combining the above estimates of \(I_1, I_2, I_3\) and the \(L^p\)-boundedness of \({\mathcal M}\), we conclude that \({{\tilde{T}}}_N^\alpha \) is a bounded operator on \(L^p({\mathbb R}^n, \omega )\) for every \(1<p<\infty \), and from \(L^1({\mathbb R}^n, \omega )\) into \(L^{1,\infty }({\mathbb R}^n, \omega )\). We should note that the constants C appeared in the above estimates all are independent with \(N=(N_1, N_2)\). Thus, the proof of the theorem is complete. \(\square \)

We shall also analyze the behavior in \(L^\infty \) and \(BMO_{{\mathcal L}}\). The space \(BMO_{{\mathcal L}}({\mathbb R}^n)\), introduced in [8], is defined as the set of functions f such that

$$\begin{aligned} \left\{ \begin{array}{ll} \displaystyle \frac{1}{|B|} \int _B\Big |f(z)- \frac{1}{|B|} \int _B f \Big | {\textrm{d}}z\le C_1, \hbox { for all } B= B_R(x): R \le \rho (x), \\ \displaystyle \frac{1}{|B|} \int _B |f| \le C_2, \hbox { for all } B= B_R(x): R >\rho (x), \end{array}\right. \end{aligned}$$

where \(\rho (x)\) is the critical radii associated with \({\mathcal L}\), see (4.1). The norm \(\Vert f\Vert _{BMO_{\mathcal L}(\mathbb {R}^n)}\) is defined as \(\min \{C_1,C_2\}\).

Theorem 12

Given \(f\in BMO_{\mathcal L}({\mathbb R}^n)\), then there exists a constant C depending only on n\(\alpha \), \(\rho \) and \(\left\| v\right\| _{l^\infty (\mathbb Z)}\) such that

$$\begin{aligned}\left\| {{\tilde{T}}}_N^\alpha f\right\| _{BMO_{\mathcal L}(\mathbb R^n)}\le C\left\| f\right\| _{BMO_{\mathcal L}(\mathbb R^n)},\end{aligned}$$

for all functions \(f\in BMO_{\mathcal L}(\mathbb R^n).\)

Proof

We first show that, if \(f\in BMO_{\mathcal L}({\mathbb R}^n)\), then \(\tilde{T}_N^\alpha f\) is finite almost everywhere. This information is contained in the lemma as follows:

Lemma 7

Given \(f\in BMO_{\mathcal L}({\mathbb R}^n)\) and any \(x_0\in {\mathbb R}^n, C_0\ge 1,\) then \({{\tilde{T}}}_N^\alpha f(x)<\infty \) at almost every \(x\in B=B(x_0, C_0\rho (x_0))\).

Proof

Let us split the function f to be

$$\begin{aligned}f=(f-f_B)\chi _{B^*}+(f-f_B)\chi _{{B^*}^c}+f_B=:f_1+f_2+f_3,\end{aligned}$$

where \(B^*=B(x_0, 2C_0\rho (x_0)).\) Since \(f\in BMO_{\mathcal L}({\mathbb R}^n)\), \(f_1\in L^1({\mathbb R}^n)\). By Theorem 11 (b), we know that \({{\tilde{T}}}_N^\alpha f_1(x)<\infty \) a.e. \(x\in B.\) For \(\tilde{T}_N^\alpha f_2\), we note that, for any \(x\in B\) and \(t>0\),

$$\begin{aligned}&{{\tilde{{\mathcal P}}}}_t^\alpha f_2(x)\le \frac{t^{2\alpha }}{4^\alpha \Gamma (\alpha )} \int _0^{+\infty } \int _{{\mathbb R}^n} \frac{e^{-(t^2+|x-y|^2)/(4 s)}}{(4\pi s)^{n/2}} f_2 (y){\textrm{d}}y \frac{{\textrm{d}}s}{s^{1+\alpha }}\nonumber \\&\le C\frac{t^{2\alpha }}{4^\alpha \Gamma (\alpha )} \int _0^{+\infty }\sum _{k=1}^{+\infty }\int _{2^kC_0\rho (x_0)<|x_0-y|\le 2^{k+1}C_0\rho (x_0)} {1\over |x-y|^{n+2\alpha '}}\left| f(y)-f_B\right| {\textrm{d}}y\ {e^{-{t^2\over 4 s}}}\frac{{\textrm{d}}s}{s^{1+\alpha -\alpha '}}\nonumber \\&\le C\frac{t^{2\alpha '}\Gamma (\alpha -\alpha ')}{4^{\alpha '}\Gamma (\alpha )} \sum _{k=1}^{+\infty }{(2^kC_0\rho (x_0))}^{-2\alpha '}{1\over {(2^kC_0\rho (x_0))}^{n}}\int _{|x_0-y|\le 2^{k+1}C_0\rho (x_0)} \left| f(y)-f_B\right| {\textrm{d}}y\\&\le C\frac{t^{2\alpha '}\Gamma (\alpha -\alpha ')}{4^{\alpha '}\Gamma (\alpha )} \sum _{k=1}^{+\infty }{(2^kC_0\rho (x_0))}^{-2\alpha '}(1+2k)\left\| f\right\| _{BMO_{\mathcal L}({\mathbb R}^n)}<\infty ,\nonumber \end{aligned}$$

where \(0<\alpha '<\alpha .\) So, \({{\tilde{{\mathcal P}}}}_t^\alpha f_2(x)\) is well defined for \(x\in B\) and \(t>0.\) Since \({{\tilde{T}}}_N^\alpha f_2(x)\) is a finite summation and \(x_0, r_0\) is arbitrary, \({{\tilde{T}}}_N^\alpha f_2(x)<\infty \) a.e. \(x\in {\mathbb R}^n.\) And, we should note that

$$\begin{aligned} \left| {{\tilde{{\mathcal P}}}}_t^\alpha f_3(x)\right|&\le \frac{t^{2\alpha }}{4^\alpha \Gamma (\alpha )} \int _0^{+\infty } \int _{{\mathbb R}^n} \frac{e^{-(t^2+|x-y|^2)/(4 s)}}{(4\pi s)^{n/2}} f_B {\textrm{d}}y \frac{{\textrm{d}}s}{s^{1+\alpha }}\\ {}&\le C \cdot f_B\le C\left\| f\right\| _{BMO_{\mathcal L}({\mathbb R}^n)}. \end{aligned}$$

So, \({{\tilde{T}}}_N^\alpha f_3(x)<\infty .\) Hence, \({{\tilde{T}}}_N^\alpha f(x)<\infty \) at almost every \(x\in B=B(x_0, C_0\rho (x_0))\). This completes the proof of the lemma. \(\square \)

Assume that \(f\in BMO_{\mathcal L}({\mathbb R}^n)\). Our goal is to show that \(\tilde{T}_N^\alpha f\in BMO_{\mathcal L}({\mathbb R}^n).\) By Theorem 10, we know that the operator \({\tilde{T}}_N^\alpha \) is a \(\gamma \)-Schrödinger-Calderón–Zygmund operator with \(\gamma =0\) appeared in [13]. By Theorem 1.2 in [13], we can prove the BMO-boundedness of \({{\tilde{T}}}_N^\alpha \) by checking a condition related with \({{\tilde{T}}}_N^\alpha 1:\)

$$\begin{aligned} \log \left( \frac{\rho (x_0)}{t}\right) \frac{1}{|B|}\int _B|\tilde{T}_N^\alpha 1(x)-({{\tilde{T}}}_N^\alpha 1)_B|~{\textrm{d}}x\le C \end{aligned}$$
(4.4)

for every ball \(B=B(x_0,t)\), \(x_0\in {\mathbb R}^n\) and \(0<t\le \tfrac{1}{2}\rho (x_0)\). In fact, since

$$\begin{aligned} |{{\tilde{T}}}_N^\alpha 1(x)-({{\tilde{T}}}_N^\alpha 1)_B|\le \frac{1}{\left| B\right| }\int _B\left| {{\tilde{T}}}_N^\alpha 1(x)-\tilde{T}_N^\alpha 1(y)\right| ~{\textrm{d}}y, \end{aligned}$$

we only need to prove that, with some \(0<\delta <\delta _0\),

$$\begin{aligned} \left| {{\tilde{T}}}_N^\alpha 1(x)-{{\tilde{T}}}_N^\alpha 1(y)\right| \le C\left( \frac{t}{\rho (x_0)}\right) ^\delta , \quad x, y \in B, \end{aligned}$$
(4.5)

and then, (4.4) follows.

We shall note that, when \(x, y \in B,\) \(\rho (x)\sim \rho (y)\sim \rho (x_0).\) First, we have

$$\begin{aligned}&\left| {{\tilde{T}}}_N^\alpha 1(x)-{{\tilde{T}}}_N^\alpha 1(y)\right| \\&=C \left| \int _{{\mathbb R}^{n}} \int _0^{+\infty }\sum _{j=N_1}^{N_2}v_j \frac{ a_{j+1}^{2\alpha }e^{-a_{j+1}^2/(4 s)}-a_j^{2\alpha } e^{-a_j^2/(4 s)} }{s^{1+\alpha }} \left( e^{-s{\mathcal L}}(x,z)-e^{-s{\mathcal L}}(y,z)\right) ~{\textrm{d}}s {\textrm{d}}z\right| \\&\le C \left| \int _{{\mathbb R}^{n}} \int _0^{4t^2}\sum _{j=N_1}^{N_2}v_j \frac{ a_{j+1}^{2\alpha }e^{-a_{j+1}^2/(4 s)}-a_j^{2\alpha } e^{-a_j^2/(4 s)} }{s^{1+\alpha }} \left( e^{-s{\mathcal L}}(x,z)-e^{-s{\mathcal L}}(y,z)\right) ~{\textrm{d}}s {\textrm{d}}z\right| \\&\quad +C \left| \int _{{\mathbb R}^{n}} \int _{4t^2}^{\rho ^2(x_0)}\sum _{j=N_1}^{N_2}v_j \frac{ a_{j+1}^{2\alpha }e^{-a_{j+1}^2/(4 s)}-a_j^{2\alpha } e^{-a_j^2/(4 s)} }{s^{1+\alpha }} \left( e^{-s{\mathcal L}}(x,z)-e^{-s{\mathcal L}}(y,z)\right) ~{\textrm{d}}s {\textrm{d}}z\right| \\&\quad +C \left| \int _{{\mathbb R}^{n}} \int _{\rho ^2(x_0)}^{+\infty }\sum _{j=N_1}^{N_2}v_j \frac{ a_{j+1}^{2\alpha }e^{-a_{j+1}^2/(4 s)}-a_j^{2\alpha } e^{-a_j^2/(4 s)} }{s^{1+\alpha }} \left( e^{-s{\mathcal L}}(x,z)-e^{-s{\mathcal L}}(y,z)\right) ~{\textrm{d}}s {\textrm{d}}z\right| \\&=:I+II+III. \end{aligned}$$

For the term I,  since \(\displaystyle \int _{{\mathbb R}^{n}}W_s(x, z){\textrm{d}}z=\int _{{\mathbb R}^{n}}W_s(y, z){\textrm{d}}z=1\) and by Lemma 4, we get

$$\begin{aligned} I&\le C \left| \int _{{\mathbb R}^{n}} \int _0^{4t^2}\sum _{j=N_1}^{N_2}v_j \frac{ a_{j+1}^{2\alpha }e^{-a_{j+1}^2/(4 s)}-a_j^{2\alpha } e^{-a_j^2/(4 s)} }{s^{1+\alpha }} \left( e^{-s{\mathcal L}}(x,z)-W_s(x,z)\right) ~{\textrm{d}}s {\textrm{d}}z\right| \\&\quad +C \left| \int _{{\mathbb R}^{n}} \int _0^{4t^2}\sum _{j=N_1}^{N_2}v_j \frac{ a_{j+1}^{2\alpha }e^{-a_{j+1}^2/(4 s)}-a_j^{2\alpha } e^{-a_j^2/(4 s)} }{s^{1+\alpha }} \left( e^{-s{\mathcal L}}(y,z)-W_s(y,z)\right) ~{\textrm{d}}s {\textrm{d}}z\right| \\&\le C\int _0^{4t^2}\sum _{j=N_1}^{N_2} \left| v_j \frac{ a_{j+1}^{2\alpha }e^{-a_{j+1}^2/(4 s)}-a_j^{2\alpha } e^{-a_j^2/(4 s)} }{s^{1+\alpha }}\right| \left( \sqrt{s} \over \rho (x)\right) ^{\delta _0}\int _{{\mathbb R}^{n}} \omega _s(x-z)~{\textrm{d}}z {\textrm{d}}s\\&\quad +C\int _0^{4t^2}\sum _{j=N_1}^{N_2} \left| v_j \frac{ a_{j+1}^{2\alpha }e^{-a_{j+1}^2/(4 s)}-a_j^{2\alpha } e^{-a_j^2/(4 s)} }{s^{1+\alpha }}\right| \left( \sqrt{s} \over \rho (y)\right) ^{\delta _0}\int _{{\mathbb R}^{n}} \omega _s(y-z)~{\textrm{d}}z {\textrm{d}}s\\&\le C\int _0^{4t^2} {1\over s} \left( \sqrt{s} \over \rho (x)\right) ^{\delta _0} {\textrm{d}}s\le C\left( t \over \rho (x_0)\right) ^{\delta _0}. \end{aligned}$$

For II,  we have

$$\begin{aligned} II&= C \Big | \int _{{\mathbb R}^{n}} \int _{4t^2}^{\rho ^2(x_0)}\sum _{j=N_1}^{N_2}v_j \frac{ a_{j+1}^{2\alpha }e^{-a_{j+1}^2/(4 s)}-a_j^{2\alpha } e^{-a_j^2/(4 s)} }{s^{1+\alpha }}\left( e^{-s{\mathcal L}}(x,z)-e^{-s{\mathcal L}}(y,z)\right) ~{\textrm{d}}s {\textrm{d}}z\Big |\\&=C\Big (\int _{\left| x-z\right| >c\rho (x)}\int _{4t^2}^{\rho ^2(x_0)}{1\over s} \left| e^{-s{\mathcal L}}(x,z)-e^{-s{\mathcal L}}(y,z)\right| ~{\textrm{d}}s {\textrm{d}}z\\&\quad +\int _{4|x-y|<\left| x-z\right| \le c\rho (x)}\int _{4t^2}^{\rho ^2(x_0)}{1\over s} \left| e^{-s{\mathcal L}}(x,z)-W_s(x, z)+W_s(y, z)-e^{-s{\mathcal L}}(y,z)\right| ~{\textrm{d}}s {\textrm{d}}z\\&\quad +\int _{\left| x-z\right| \le 4|x-y|}\int _{4t^2}^{\rho ^2(x_0)}{1\over s} \left| e^{-s{\mathcal L}}(x,z)-W_s(x, z)+W_s(y, z)-e^{-s{\mathcal L}}(y,z)\right| ~{\textrm{d}}s {\textrm{d}}z\Big )\\&=:C\left( II_1+II_2+II_3\right) . \end{aligned}$$

Since \(\left| x-y\right| \le 2t\le \sqrt{s},\) by Lemma 5 we have

$$\begin{aligned} II_1&\le C\int _{\left| x-z\right|>c\rho (x)}\int _{4t^2}^{\rho ^2(x_0)}{1\over s} \left( \left| x-y\right| \over \sqrt{s}\right) ^\delta \omega _s(x-z)~{\textrm{d}}s{\textrm{d}}z\\&\le C\int _{\left| x-z\right|>c\rho (x)}{\left| x-y\right| ^\delta \over \left| x-z\right| ^{2+\delta }} \int _{4t^2}^{\rho ^2(x_0)}\omega _s(x-z)~{\textrm{d}}s{\textrm{d}}z\\&\le C\int _{\left| x-z\right|>c\rho (x)}{\left| x-y\right| ^\delta \over \left| x-z\right| ^{n+2+\delta }} \int _{4t^2}^{\rho ^2(x_0)}1~{\textrm{d}}s{\textrm{d}}z\\&\le C\int _{\left| x-z\right| >c\rho (x)}{\left| x-y\right| ^\delta \over \left| x-z\right| ^{n+2+\delta }} \rho ^2(x_0){\textrm{d}}z\\&\le C\left( t\over \rho (x_0)\right) ^\delta . \end{aligned}$$

For the term \(II_2\), in this case, \(|x-y|< c\rho (x)\) and \(\displaystyle |x-y|<{|x-z|\over 4}\). By Lemma 6 we get

$$\begin{aligned} II_2&\le C\int _{4|x-y|<\left| x-z\right| \le c\rho (x)}\int _{4t^2}^{\rho ^2(x_0)}{1\over s} \left( {\left| x-y\right| \over \rho (z)}\right) ^\delta \omega _s(x-z)~{\textrm{d}}s {\textrm{d}}z\\&\le C\int _{4|x-y|<\left| x-z\right| \le c\rho (x)}\left( {\left| x-y\right| \over \rho (z)}\right) ^\delta {1\over |x-z|^{n+2}}\int _{4t^2}^{\rho ^2(x_0)}1~{\textrm{d}}s {\textrm{d}}z\\&\le C\left( {\left| x-y\right| \over \rho (x_0)}\right) ^\delta \int _{4|x-y|<\left| x-z\right| \le c\rho (x)} {\rho ^2(x_0)\over |x-z|^{n+2}} {\textrm{d}}z\\&\le C\left( {\left| x-y\right| \over \rho (x_0)}\right) ^\delta \le C \left( {t\over \rho (x_0)}\right) ^{\delta }. \end{aligned}$$

For the term \(II_3,\) by Lemma 4, we have

$$\begin{aligned} II_3&\le C\int _{\left| x-z\right| \le 4|x-y|}\int _{4t^2}^{\rho ^2(x_0)}{1\over s} \left[ {\left( {\sqrt{s}\over \rho (x)}\right) }^{\delta _0}\omega _s(x-z)+{\left( {\sqrt{s}\over \rho (y)}\right) }^{\delta _0}\omega _s(y-z)\right] ~{\textrm{d}}s {\textrm{d}}z\\&\le C\Big (\int _{\left| x-z\right| \le 4|x-y|}\int _{4t^2}^{\rho ^2(x_0)}{1\over s}{\left( {\sqrt{s}\over \rho (x)}\right) }^{\delta _0}\omega _s(x-z)~{\textrm{d}}s{\textrm{d}}z\\&\quad +\int _{\left| y-z\right| \le 5|x-y|}\int _{4t^2}^{\rho ^2(x_0)}{1\over s}{\left( {\sqrt{s}\over \rho (y)}\right) }^{\delta _0}\omega _s(y-z)~{\textrm{d}}s {\textrm{d}}z\Big )\\&\le C \int _{4t^2}^{\rho ^2(x_0)} \int _{\left| \xi \right| \le 5{|x-y|\over \sqrt{s}}}{1\over s} {\left( {\sqrt{s}\over \rho (x)}\right) }^{\delta _0}\omega _s(\xi )~ d\xi {\textrm{d}}s\\&\le C \int _{4t^2}^{+\infty } {1\over s} {\left( {\sqrt{s}\over \rho (x_0)}\right) }^{\delta _0}\left( {|x-y|\over \sqrt{s}}\right) ^n {\textrm{d}}s\\&\le C {{|x-y|^n \over \rho (x_0)^{\delta _0}}}~t^{\delta _0-n}\le C \left( {t\over \rho (x_0)}\right) ^{\delta _0}. \end{aligned}$$

Then, we get

$$\begin{aligned} II\le C\left( {t\over \rho (x_0)}\right) ^{\delta } \end{aligned}$$

with some \(0<\delta <\delta _0.\)

We shall treat the latest term III. In this case, since \(s\ge \rho ^2(x_0)>4t^2\), then \(\sqrt{s}>2t>|x-y|\). By Lemma 5, we have, for \(0<\delta <\delta _0,\)

$$\begin{aligned}{} & {} III\le C \int _{\rho ^2(x_0)}^{+\infty }{1\over s} \left( {|x-y|\over \sqrt{s}}\right) ^\delta \int _{{\mathbb R}^{n}}\omega _s(x-z) {\textrm{d}}z~{\textrm{d}}s \\{} & {} \quad \le C \int _{\rho ^2(x_0)}^{+\infty }{1\over s} \left( {|x-y|\over \sqrt{s}}\right) ^\delta ~{\textrm{d}}s \le C\left( {|x-y|\over \rho (x_0)}\right) ^\delta \le C\left( {t\over \rho (x_0)}\right) ^\delta . \end{aligned}$$

Combining the above estimates for III and III,  we have proved (4.5). Hence, we get the estimation (4.4) and the BMO-boundedness of \({{\tilde{T}}}_N^\alpha .\) This completes the proof of Theorem 12. \(\square \)

As in the Laplacian case, we also can consider the maximal operator

$$\begin{aligned} {{\tilde{T}}}^*f(x)=\sup _N \left| {{\tilde{T}}}_N^\alpha f(x)\right| , \quad x\in {\mathbb R}^n, \end{aligned}$$

where the supremum are taken over all \(N=(N_1,N_2)\in {\mathbb Z}^2\) with \(N_1< N_2\).

Now we present our results as follows:

Theorem 13

  1. (a)

    For any \(1<p<\infty \) and \(\omega \in A_p\), there exists a constant C depending on \(n, p, \rho , \alpha \) and \(\left\| v\right\| _{l^\infty (\mathbb Z)}\) such that

    $$\begin{aligned}\left\| {{\tilde{T}}}^*f\right\| _{L^p(\mathbb R^n, \omega )}\le C\left\| f\right\| _{L^p(\mathbb R^n, \omega )},\end{aligned}$$

    for all functions \(f\in L^p(\mathbb R^n, \omega ).\)

  2. (b)

    For any \(\lambda >0\) and \(\omega \in A_1\), there exists a constant C depending on \(n, \rho , \alpha \) and \(\left\| v\right\| _{l^\infty (\mathbb Z)}\) such that

    $$\begin{aligned}\omega \left( {\{x\in {\mathbb R}^n:\left| {{\tilde{T}}}^*f(x)\right| >\lambda \}}\right) \le C\frac{1}{\lambda }\left\| f\right\| _{L^1(\mathbb R^n, \omega )},\end{aligned}$$

    for all functions \(f\in L^1(\mathbb R^n, \omega ).\)

  3. (c)

    There exists a constant C depending on \(n, \rho \), \(\alpha \) and \(\left\| v\right\| _{l^\infty (\mathbb Z)}\) such that

    $$\begin{aligned}\left\| {{\tilde{T}}}^*f\right\| _{BMO_{\mathcal L}(\mathbb R^n)}\le C\left\| f\right\| _{L^\infty (\mathbb R^n)},\end{aligned}$$

    for any \(f\in L^\infty ({\mathbb R}^n)\).

  4. (d)

    There exists a constant C depending on \(n, \rho \), \(\alpha \) and \(\left\| v\right\| _{l^\infty (\mathbb Z)}\) such that

    $$\begin{aligned} \left\| {{\tilde{T}}}^*f\right\| _{ BMO_{\mathcal L}(\mathbb R^n)}\le C\left\| f\right\| _{BMO_{\mathcal L}(\mathbb R^n)}, \end{aligned}$$

    for all functions \(f\in BMO_{\mathcal L}({\mathbb R}^n).\)

With a similar argument as in the proof of Theorem 1, we can prove a Cotlar’s type inequality in the Schrödinger setting. And then, all the statements in Theorem 13 can be gotten just with minor changes. We omit the proof at here.